Вы находитесь на странице: 1из 73

TEST I - Foundation of Professional Nursing Practice 1.

The nurse In-charge in labor and delivery unit administered a dose of terbutaline to a client without checking the clients pulseThe standard that would be used to determine if the nurse was negligent is: a. The physicians orders b. The action of a clinical nurse specialist who is recognized expert in the field. c. The statement in the drug literature about administration of terbutaline. d. The actions of a reasonably prudent nurse with similar education and experience. 2. Nurse Trish is caring for a female client with a history of GI bleeding, sickle cell disease, and a platelet count of 80,000mm3The female client is dehydrated and receiving dextrose 5% in halfnormal saline solution at 150 ml/hr. The client complains of severe bone pain and is scheduled to receive a dose of morphine sulfate. In administering the medication, Nurse Trish should avoid which route? a. I.V b. I.M c. Oral d. S.C 3. Dr. Garcia writes the following order for the client who has been recently admitted Digoxin mg POonce dailyTo prevent a dosage error, how should the nurse document this order onto the medication administration record? a. Digoxin mg P.O.once daily b. Digoxin mg P.O.once daily

c. Digoxin mg POonce daily d. Digoxin mg POonce daily 4. A newly admitted female client was diagnosed with deep vein thrombosis. Which nursing diagnosis should receive the highest priority? a. Ineffective peripheral tissue perfusion related to venous congestion. b. Risk for injury related to edema. c. Excess fluid volume related to peripheral vascular disease. d. Impaired gas exchange related to increased blood flow. 5. Nurse Betty is assigned to the following clients. The client that the nurse would see first after endorsement? a. A 34 year-old post-operative appendectomy client of five hours who is complaining of pain. b. A 44 year-old myocardial infarction (MI) client who is complaining of nausea. c. A 26 year-old client admitted for dehydration whose intravenous (IV) has infiltrated. d. A 63 year-old post operative abdominal hysterectomy client of three days whose incisional dressing is saturated with serosanguinous fluid. 6. Nurse Gail places a client in a four-point restraint following orders from the physician. The client care plan should include: a. Assess temperature frequently. b. Provide diversional activities. c. Check circulation every 15-30 minutes. d. Socialize with other patients once a shift. 7. A male client who has severe burns is receiving

H2 receptor antagonist therapy. The nurse Incharge knows the purpose of this therapy is to: a. Prevent stress ulcer b. Block prostaglandin synthesis c. Facilitate protein synthesis. d. Enhance gas exchange 8. The doctor orders hourly urine output measurement for a postoperative male client. The nurse Trish records the following amounts of output for 2 consecutive hours: 8 a.m.: 50 ml; 9 a.m.: 60 ml. Based on these amounts, which action should the nurse take? a. Increase the I.V. fluid infusion rate b. Irrigate the indwelling urinary catheter c. Notify the physician d. Continue to monitor and record hourly urine output 9. Tony, a basketball player twist his right ankle while playing on the court and seeks care for ankle pain and swelling. After the nurse applies ice to the ankle for 30 minutes, which statement by Tony suggests that ice application has been effective? a. My ankle looks less swollen now b. My ankle feels warm c. My ankle appears redder now d. I need something stronger for pain relief 10. The physician prescribes a loop diuretic for a client. When administering this drug, the nurse anticipates that the client may develop which electrolyte imbalance? a. Hypernatremia b. Hyperkalemia c. Hypokalemia

d. Hypervolemia 11. She finds out that some managers have benevolent-authoritative style of management. Which of the following behaviors will she exhibit most likely? a. Have condescending trust and confidence in their subordinates. b. Gives economic and ego awards. c. Communicates downward to staffs. d. Allows decision making among subordinates. 12. Nurse Amy is aware that the following is true about functional nursing a. Provides continuous, coordinated and comprehensive nursing services. b. One-to-one nurse patient ratio. c. Emphasize the use of group collaboration. d. Concentrates on tasks and activities. 13. Which type of medication order might read "Vitamin K 10 mg I.M. daily 3 days?" a. Single order b. Standard written order c. Standing order d. Stat order 14. A female client with a fecal impaction frequently exhibits which clinical manifestation? a. Increased appetite b. Loss of urge to defecate c. Hard, brown, formed stools d. Liquid or semi-liquid stools 15. Nurse Linda prepares to perform an otoscopic examination on a female client. For proper visualization, the nurse should position the client's ear by: a. Pulling the lobule down and back

b. Pulling the helix up and forward c. Pulling the helix up and back d. Pulling the lobule down and forward 16. Which instruction should nurse Tom give to a male client who is having external radiation therapy: a. Protect the irritated skin from sunlight. b. Eat 3 to 4 hours before treatment. c. Wash the skin over regularly. d. Apply lotion or oil to the radiated area when it is red or sore. 17. In assisting a female client for immediate surgery, the nurse In-charge is aware that she should: a. Encourage the client to void following preoperative medication. b. Explore the clients fears and anxieties about the surgery. c. Assist the client in removing dentures and nail polish. d. Encourage the client to drink water prior to surgery. 18. A male client is admitted and diagnosed with acute pancreatitis after a holiday celebration of excessive food and alcohol. Which assessment finding reflects this diagnosis? a. Blood pressure above normal range. b. Presence of crackles in both lung fields. c. Hyperactive bowel sounds d. Sudden onset of continuous epigastric and back pain. 19. Which dietary guidelines are important for nurse Oliver to implement in caring for the client with burns?

a. Provide high-fiber, high-fat diet b. Provide high-protein, high-carbohydrate diet. c. Monitor intake to prevent weight gain. d. Provide ice chips or water intake. 20. Nurse Hazel will administer a unit of whole blood, which priority information should the nurse have about the client? a. Blood pressure and pulse rate. b. Height and weight. c. Calcium and potassium levels d. Hgb and Hct levels. 21. Nurse Michelle witnesses a female client sustain a fall and suspects that the leg may be broken. The nurse takes which priority action? a. Takes a set of vital signs. b. Call the radiology department for X-ray. c. Reassure the client that everything will be alright. d. Immobilize the leg before moving the client. 22. A male client is being transferred to the nursing unit for admission after receiving a radium implant for bladder cancer. The nurse in-charge would take which priority action in the care of this client? a. Place client on reverse isolation. b. Admit the client into a private room. c. Encourage the client to take frequent rest periods. d. Encourage family and friends to visit. 23. A newly admitted female client was diagnosed with agranulocytosis. The nurse formulates which priority nursing diagnosis? a. Constipation

b. Diarrhea c. Risk for infection d. Deficient knowledge 24. A male client is receiving total parenteral nutrition suddenly demonstrates signs and symptoms of an air embolism. What is the priority action by the nurse? a. Notify the physician. b. Place the client on the left side in the Trendelenburg position. c. Place the client in high-Fowlers position. d. Stop the total parenteral nutrition. 25. Nurse May attends an educational conference on leadership styles. The nurse is sitting with a nurse employed at a large trauma center who states that the leadership style at the trauma center is task-oriented and directive. The nurse determines that the leadership style used at the trauma center is: a. Autocratic. b. Laissez-faire. c. Democratic. d. Situational 26. The physician orders DS 500 cc with KCl 10 mEq/liter at 30 cc/hr. The nurse in-charge is going to hang a 500 cc bag. KCl is supplied 20 mEqccHow many ccs of KCl will be added to the IV solution? a. .5 cc b. 5 cc c. 1.5 cc d. 2.5 cc 27. A child of 10 years old is to receive 400 cc of IV fluid in an 8 hour shift. The IV drip factor is 60.

The IV rate that will deliver this amount is: a. 50 cc/ hour b. 55 cc/ hour c. 24 cc/ hour d. 66 cc/ hour 28. The nurse is aware that the most important nursing action when a client returns from surgery is: a. Assess the IV for type of fluid and rate of flow. b. Assess the client for presence of pain. c. Assess the Foley catheter for patency and urine output d. Assess the dressing for drainage. 29. Which of the following vital sign assessments that may indicate cardiogenic shock after myocardial infarction? a. BP ,06/08Pulse 011irregular b. BP ,05/09Pulse 05regular c. BP ,08/031Pulse 001regular d. BP ,001/081Pulse 09irregular 30. Which is the most appropriate nursing action in obtaining a blood pressure measurement? a. Take the proper equipment, place the client in a comfortable position, and record the appropriate information in the clients chart b. Measure the clients arm, if you are not sure of the size of cuff to use. c. Have the client recline or sit comfortably in a chair with the forearm at the level of the heart. d. Document the measurement, which extremity was used, and the position that the client was in during the measurement.

31. Asking the questions to determine if the person understands the health teaching provided by the nurse would be included during which step of the nursing process? a. Assessment b. Evaluation c. Implementation d. Planning and goals 32. Which of the following item is considered the single most important factor in assisting the health professional in arriving at a diagnosis or determining the persons needs? a. Diagnostic test results b. Biographical date c. History of present illness d. Physical examination 33. In preventing the development of an external rotation deformity of the hip in a client who must remain in bed for any period of time, the most appropriate nursing action would be to use: a. Trochanter roll extending from the crest of the ileum to the mid-thigh. b. Pillows under the lower legs. c. Footboard d. Hip-abductor pillow 34. Which stage of pressure ulcer development does the ulcer extend into the subcutaneous tissue? a. Stage I b. Stage II c. Stage III d. Stage IV 35. When the method of wound healing is one in

which wound edges are not surgically approximated and integumentary continuity is restored by granulations, the wound healing is termed a. Second intention healing b. Primary intention healing c. Third intention healing d. First intention healing 36. An 80-year-old male client is admitted to the hospital with a diagnosis of pneumonia. Nurse Oliver learns that the client lives alone and hasnt been eating or drinkingWhen assessing him for dehydration, nurse Oliver would expect to find: a. Hypothermia b. Hypertension c. Distended neck veins d. Tachycardia 37. The physician prescribes meperidine (Demerol), 75 mg I.M. every 4 hours as needed, to control a clients postoperative painThe package insert is Meperidine, mgmlHow many millilitres of meperidine should the client receive? a. 0.75 b. 0.6 c. 0.5 d. 0.25 38. A male client with diabetes mellitus is receiving insulin. Which statement correctly describes an insulin unit? a. Its a common measurement in the metric system. b. Its the basis for solids in the avoirdupois

system. c. Its the smallest measurement in the apothecary system. d. Its a measure of effect, not a standard measure of weight or quantity. 39. Nurse Oliver measures a clients temperature at 102 F. What is the equivalent Centigrade temperature? a. 40.1 C b. 38.9 C c. 48 C d. 38 C 40. The nurse is assessing a 48-year-old client who has come to the physicians office for his annual physical exam. One of the first physical signs of aging is: a. Accepting limitations while developing assets. b. Increasing loss of muscle tone. c. Failing eyesight, especially close vision. d. Having more frequent aches and pains. 41. The physician inserts a chest tube into a female client to treat a pneumothorax. The tube is connected to water-seal drainage. The nurse incharge can prevent chest tube air leaks by: a. Checking and taping all connections. b. Checking patency of the chest tube. c. Keeping the head of the bed slightly elevated. d. Keeping the chest drainage system below the level of the chest. 42. Nurse Trish must verify the clients identity before administering medication. She is aware that the safest way to verify identity is to: a. Check the clients identification band b. Ask the client to state his name.

c. State the clients name out loud and wait a client to repeat it. d. Check the room number and the clients name on the bed. 43. The physician orders dextrose 5 % in water, 1,000 ml to be infused over 8 hours. The I.V. tubing delivers 15 drops/ml. Nurse John should run the I.V. infusion at a rate of: a. 30 drops/minute b. 32 drops/minute c. 20 drops/minute d. 18 drops/minute 44. If a central venous catheter becomes disconnected accidentally, what should the nurse in-charge do immediately? a. Clamp the catheter b. Call another nurse c. Call the physician d. Apply a dry sterile dressing to the site. 45. A female client was recently admitted. She has fever, weight loss, and watery diarrhea is being admitted to the facility. While assessing the client, Nurse Hazel inspects the clients abdomen and notice that it is slightly concave. Additional assessment should proceed in which order: a. Palpation, auscultation, and percussion. b. Percussion, palpation, and auscultation. c. Palpation, percussion, and auscultation. d. Auscultation, percussion, and palpation. 46. Nurse Betty is assessing tactile fremitus in a client with pneumonia. For this examination, nurse Betty should use the: a. Fingertips b. Finger pads

c. Dorsal surface of the hand d. Ulnar surface of the hand 47. Which type of evaluation occurs continuously throughout the teaching and learning process? a. Summative b. Informative c. Formative d. Retrospective 48. A 45 year old client, has no family history of breast cancer or other risk factors for this disease. Nurse John should instruct her to have mammogram how often? a. Twice per year b. Once per year c. Every 2 years d. Once, to establish baseline 49. A male client has the following arterial blood gas values: pH 7.30; Pao2 89 mmHg; Paco2 50 mmHg; and HCO3 26mEq/L. Based on these values, Nurse Patricia should expect which condition? a. Respiratory acidosis b. Respiratory alkalosis c. Metabolic acidosis d. Metabolic alkalosis 50. Nurse Len refers a female client with terminal cancer to a local hospice. What is the goal of this referral? a. To help the client find appropriate treatment options. b. To provide support for the client and family in coping with terminal illness. c. To ensure that the client gets counseling regarding health care costs.

d. To teach the client and family about cancer and its treatment. 51. When caring for a male client with a 3-cm stage I pressure ulcer on the coccyx, which of the following actions can the nurse institute independently? a. Massaging the area with an astringent every 2 hours. b. Applying an antibiotic cream to the area three times per day. c. Using normal saline solution to clean the ulcer and applying a protective dressing as necessary. d. Using a povidone-iodine wash on the ulceration three times per day. 52. Nurse Oliver must apply an elastic bandage to a clients ankle and calfHe should apply the bandage beginning at the clients a. Knee b. Ankle c. Lower thigh d. Foot 53. A 10 year old child with type 1 diabetes develops diabetic ketoacidosis and receives a continuous insulin infusion. Which condition represents the greatest risk to this child? a. Hypernatremia b. Hypokalemia c. Hyperphosphatemia d. Hypercalcemia 54. Nurse Len is administering sublingual nitrglycerin (Nitrostat) to the newly admitted client.

Immediately afterward, the client may experience: a. Throbbing headache or dizziness b. Nervousness or paresthesia. c. Drowsiness or blurred vision. d. Tinnitus or diplopia. 55. Nurse Michelle hears the alarm sound on the telemetry monitor. The nurse quickly looks at the monitor and notes that a client is in a ventricular tachycardia. The nurse rushes to the clients room Upon reaching the clients bedside, the nurse would take which action first? a. Prepare for cardioversion b. Prepare to defibrillate the client c. Call a code d. Check the clients level of consciousness 56. Nurse Hazel is preparing to ambulate a female client. The best and the safest position for the nurse in assisting the client is to stand: a. On the unaffected side of the client. b. On the affected side of the client. c. In front of the client. d. Behind the client. 57. Nurse Janah is monitoring the ongoing care given to the potential organ donor who has been diagnosed with brain death. The nurse determines that the standard of care had been maintained if which of the following data is observed? a. Urine output: 45 ml/hr b. Capillary refill: 5 seconds c. Serum pH: 7.32 d. Blood pressure: 90/48 mmHg 58. Nurse Amy has an order to obtain a urinalysis from a male client with an indwelling urinary

catheter. The nurse avoids which of the following, which contaminate the specimen? a. Wiping the port with an alcohol swab before inserting the syringe. b. Aspirating a sample from the port on the drainage bag. c. Clamping the tubing of the drainage bag. d. Obtaining the specimen from the urinary drainage bag. 59. Nurse Meredith is in the process of giving a client a bed bath. In the middle of the procedure, the unit secretary calls the nurse on the intercom to tell the nurse that there is an emergency phone call. The appropriate nursing action is to: a. Immediately walk out of the clients room and answer the phone call. b. Cover the client, place the call light within reach, and answer the phone call. c. Finish the bed bath before answering the phone call. d. Leave the clients door open so the client can be monitored and the nurse can answer the phone call. 60. Nurse Janah is collecting a sputum specimen for culture and sensitivity testing from a client who has a productive cough. Nurse Janah plans to implement which intervention to obtain the specimen? a. Ask the client to expectorate a small amount of sputum into the emesis basin. b. Ask the client to obtain the specimen after breakfast.

c. Use a sterile plastic container for obtaining the specimen. d. Provide tissues for expectoration and obtaining the specimen. 61. Nurse Ron is observing a male client using a walker. The nurse determines that the client is using the walker correctly if the client: a. Puts all the four points of the walker flat on the floor, puts weight on the hand pieces, and then walks into it. b. Puts weight on the hand pieces, moves the walker forward, and then walks into it. c. Puts weight on the hand pieces, slides the walker forward, and then walks into it. d. Walks into the walker, puts weight on the hand pieces, and then puts all four points of the walker flat on the floor. 62. Nurse Amy has documented an entry regarding client care in the clients medical record When checking the entry, the nurse realizes that incorrect information was documented. How does the nurse correct this error? a. Erases the error and writes in the correct information. b. Uses correction fluid to cover up the incorrect information and writes in the correct information. c. Draws one line to cross out the incorrect information and then initials the change. d. Covers up the incorrect information completely using a black pen and writes in the correct information

63. Nurse Ron is assisting with transferring a client from the operating room table to a stretcher. To provide safety to the client, the nurse should: a. Moves the client rapidly from the table to the stretcher. b. Uncovers the client completely before transferring to the stretcher. c. Secures the client safety belts after transferring to the stretcher. d. Instructs the client to move self from the table to the stretcher. 64. Nurse Myrna is providing instructions to a nursing assistant assigned to give a bed bath to a client who is on contact precautions. Nurse Myrna instructs the nursing assistant to use which of the following protective items when giving bed bath? a. Gown and goggles b. Gown and gloves c. Gloves and shoe protectors d. Gloves and goggles 65. Nurse Oliver is caring for a client with impaired mobility that occurred as a result of a stroke. The client has right sided arm and leg weakness. The nurse would suggest that the client use which of the following assistive devices that would provide the best stability for ambulating? a. Crutches b. Single straight-legged cane c. Quad cane d. Walker 66. A male client with a right pleural effusion noted on a chest X-ray is being prepared for thoracentesis. The client experiences severe

dizziness when sitting upright. To provide a safe environment, the nurse assists the client to which position for the procedure? a. Prone with head turned toward the side supported by a pillow. b. Sims position with the head of the bed flat. c. Right side-lying with the head of the bed elevated 45 degrees. d. Left side-lying with the head of the bed elevated 45 degrees. 67. Nurse John develops methods for data gathering. Which of the following criteria of a good instrument refers to the ability of the instrument to yield the same results upon its repeated administration? a. Validity b. Specificity c. Sensitivity d. Reliability 68. Harry knows that he has to protect the rights of human research subjects. Which of the following actions of Harry ensures anonymity? a. Keep the identities of the subject secret b. Obtain informed consent c. Provide equal treatment to all the subjects of the study. d. Release findings only to the participants of the study 69. Patients refusal to divulge information is a limitation because it is beyond the control of TifannyWhat type of research is appropriate for this study? a. Descriptive- correlational b. Experiment

c. Quasi-experiment d. Historical 70. Nurse Ronald is aware that the best tool for data gathering is? a. Interview schedule b. Questionnaire c. Use of laboratory data d. Observation 71. Monica is aware that there are times when only manipulation of study variables is possible and the elements of control or randomization are not attendant. Which type of research is referred to this? a. Field study b. Quasi-experiment c. Solomon-Four group design d. Post-test only design 72. Cherry notes down ideas that were derived from the description of an investigation written by the person who conducted it. Which type of reference source refers to this? a. Footnote b. Bibliography c. Primary source d. Endnotes 73. When Nurse Trish is providing care to his patient, she must remember that her duty is bound not to do doing any action that will cause the patient harm. This is the meaning of the bioethical principle: a. Non-maleficence b. Beneficence c. Justice d. Solidarity

74. When a nurse in-charge causes an injury to a female patient and the injury caused becomes the proof of the negligent act, the presence of the injury is said to exemplify the principle of: a. Force majeure b. Respondeat superior c. Res ipsa loquitor d. Holdover doctrine 75. Nurse Myrna is aware that the Board of Nursing has quasi-judicial power. An example of this power is: a. The Board can issue rules and regulations that will govern the practice of nursing b. The Board can investigate violations of the nursing law and code of ethics c. The Board can visit a school applying for a permit in collaboration with CHED d. The Board prepares the board examinations 76. When the license of nurse Krina is revoked, it means that she: a. Is no longer allowed to practice the profession for the rest of her life b. Will never have her/his license re-issued since it has been revoked c. May apply for re-issuance of his/her license based on certain conditions stipulated in RA 9173 d. Will remain unable to practice professional nursing 77. Ronald plans to conduct a research on the use of a new method of pain assessment scale. Which of the following is the second step in the conceptualizing phase of the research process? a. Formulating the research hypothesis

b. Review related literature c. Formulating and delimiting the research problem d. Design the theoretical and conceptual framework 78. The leader of the study knows that certain patients who are in a specialized research setting tend to respond psychologically to the conditions of the study. This referred to as : a. Cause and effect b. Hawthorne effect c. Halo effect d. Horns effect 79. Mary finally decides to use judgment sampling on her research. Which of the following actions of is correct? a. Plans to include whoever is there during his study. b. Determines the different nationality of patients frequently admitted and decides to get representations samples from each. c. Assigns numbers for each of the patients, place these in a fishbowl and draw 10 from it. d. Decides to get 20 samples from the admitted patients 80. The nursing theorist who developed transcultural nursing theory is: a. Florence Nightingale b. Madeleine Leininger c. Albert Moore d. Sr. Callista Roy 81. Marion is aware that the sampling method that

gives equal chance to all units in the population to get picked is: a. Random b. Accidental c. Quota d. Judgment 82. John plans to use a Likert Scale to his study to determine the: a. Degree of agreement and disagreement b. Compliance to expected standards c. Level of satisfaction d. Degree of acceptance 83. Which of the following theory addresses the four modes of adaptation? a. Madeleine Leininger b. Sr. Callista Roy c. Florence Nightingale d. Jean Watson 84. Ms. Garcia is responsible to the number of personnel reporting to her. This principle refers to: a. Span of control b. Unity of command c. Downward communication d. Leader 85. Ensuring that there is an informed consent on the part of the patient before a surgery is done, illustrates the bioethical principle of: a. Beneficence b. Autonomy c. Veracity d. Non-maleficence 86. Nurse Reese is teaching a female client with peripheral vascular disease about foot care;

Nurse Reese should include which instruction? a. Avoid wearing cotton socks. b. Avoid using a nail clipper to cut toenails. c. Avoid wearing canvas shoes. d. Avoid using cornstarch on feet. 87. A client is admitted with multiple pressure ulcers. When developing the client's diet plan, the nurse should include: a. Fresh orange slices b. Steamed broccoli c. Ice cream d. Ground beef patties 88. The nurse prepares to administer a cleansing enema. What is the most common client position used for this procedure? a. Lithotomy b. Supine c. Prone d. Simsleft lateral 89. Nurse Marian is preparing to administer a blood transfusion. Which action should the nurse take first? a. Arrange for typing and cross matching of the clients blood b. Compare the clients identification wristband with the tag on the unit of blood. c. Start an I.V. infusion of normal saline solution. d. Measure the clients vital signs 90. A 65 years old male client requests his medication at 9 p.m. instead of 10 p.m. so that he can go to sleep earlier. Which type of nursing intervention is required? a. Independent

b. Dependent c. Interdependent d. Intradependent 91. A female client is to be discharged from an acute care facility after treatment for right leg thrombophlebitis. The Nurse Betty notes that the client's leg is pain-free, without redness or edema. The nurse's actions reflect which step of the nursing process? a. Assessment b. Diagnosis c. Implementation d. Evaluation 92. Nursing care for a female client includes removing elastic stockings once per day. The Nurse Betty is aware that the rationale for this intervention? a. To increase blood flow to the heart b. To observe the lower extremities c. To allow the leg muscles to stretch and relax d. To permit veins in the legs to fill with blood. 93. Which nursing intervention takes highest priority when caring for a newly admitted client who's receiving a blood transfusion? a. Instructing the client to report any itching, swelling, or dyspnea. b. Informing the client that the transfusion usually take 1 to 2 hours. c. Documenting blood administration in the client care record. d. Assessing the clients vital signs when the transfusion ends.

94. A male client complains of abdominal discomfort and nausea while receiving tube feedings. Which intervention is most appropriate for this problem? a. Give the feedings at room temperature. b. Decrease the rate of feedings and the concentration of the formula. c. Place the client in semi-Fowler's position while feeding. d. Change the feeding container every 12 hours. 95. Nurse Patricia is reconstituting a powdered medication in a vial. After adding the solution to the powder, she nurse should: a. Do nothing. b. Invert the vial and let it stand for 3 to 5 minutes. c. Shake the vial vigorously. d. Roll the vial gently between the palms. 96. Which intervention should the nurse Trish use when administering oxygen by face mask to a female client? a. Secure the elastic band tightly around the client's head. b. Assist the client to the semi-Fowler position if possible. c. Apply the face mask from the client's chin up over the nose. d. Loosen the connectors between the oxygen equipment and humidifier. 97. The maximum transfusion time for a unit of packed red blood cells (RBCs) is: a. 6 hours b. 4 hours

c. 3 hours d. 2 hours 98. Nurse Monique is monitoring the effectiveness of a client's drug therapy. When should the nurse Monique obtain a blood sample to measure the trough drug level? a. 1 hour before administering the next dose. b. Immediately before administering the next dose. c. Immediately after administering the next dose. d. 30 minutes after administering the next dose. 99. Nurse May is aware that the main advantage of using a floor stock system is: a. The nurse can implement medication orders quickly. b. The nurse receives input from the pharmacist. c. The system minimizes transcription errors. d. The system reinforces accurate calculations. 100. Nurse Oliver is assessing a client's abdomen. Which finding should the nurse report as abnormal? a. Dullness over the liver. b. Bowel sounds occurring every 10 seconds. c. Shifting dullness over the abdomen. d. Vascular sounds heard over the renal arteries.

pregnancy test was positive but she began to have mild cramps and is now having moderate vaginal bleeding. During the physical examination of the client, the nurse notes that May has a dilated cervix. The nurse determines that May is experiencing which type of abortion? a. Inevitable b. Incomplete c. Threatened d. Septic 2. Nurse Reese is reviewing the record of a pregnant client for her first prenatal visit. Which of the following data, if noted on the clients record, would alert the nurse that the client is at risk for a spontaneous abortion? a. Age 36 years b. History of syphilis c. History of genital herpes d. History of diabetes mellitus 3. Nurse Hazel is preparing to care for a client who is newly admitted to the hospital with a possible diagnosis of ectopic pregnancy. Nurse Hazel develops a plan of care for the client and determines that which of the following nursing actions is the priority? a. Monitoring weight b. Assessing for edema c. Monitoring apical pulse

TEST II - Community Health Nursing and Care of the Mother and Child 1. May arrives at the health care clinic and tells the nurse that her last menstrual period was 9 weeks ago. She also tells the nurse that a home

d. Monitoring temperature 4. Nurse Oliver is teaching a diabetic pregnant client about nutrition and insulin needs during pregnancy. The nurse determines that the client understands dietary and insulin needs if the client states that the second half of pregnancy

requires: a. Decreased caloric intake b. Increased caloric intake c. Decreased Insulin d. Increase Insulin 5. Nurse Michelle is assessing a 24 year old client with a diagnosis of hydatidiform mole. She is aware that one of the following is unassociated with this condition? a. Excessive fetal activity. b. Larger than normal uterus for gestational age. c. Vaginal bleeding d. Elevated levels of human chorionic gonadotropin. 6. A pregnant client is receiving magnesium sulfate for severe pregnancy induced hypertension (PIH). The clinical findings that would warrant use of the antidote , calcium gluconate is: a. Urinary output 90 cc in 2 hours. b. Absent patellar reflexes. c. Rapid respiratory rate above 40/min. d. Rapid rise in blood pressure. 7. During vaginal examination of Janah who is in labor, the presenting part is at station plus two. Nurse, correctly interprets it as: a. Presenting part is 2 cm above the plane of the ischial spines. b. Biparietal diameter is at the level of the ischial spines. c. Presenting part in 2 cm below the plane of the ischial spines. d. Biparietal diameter is 2 cm above the ischial spines.

8. A pregnant client is receiving oxytocin (Pitocin) for induction of labor. A condition that warrant the nurse in-charge to discontinue I.V. infusion of Pitocin is: a. Contractions every 1 minutes lasting 70-80 seconds. b. Maternal temperature 101.2 c. Early decelerations in the fetal heart rate. d. Fetal heart rate baseline 140-160 bpm. 9. Calcium gluconate is being administered to a client with pregnancy induced hypertension (PIH). A nursing action that must be initiated as the plan of care throughout injection of the drug is: a. Ventilator assistance b. CVP readings c. EKG tracings d. Continuous CPR 10. A trial for vaginal delivery after an earlier caesarean, would likely to be given to a gravida, who had: a. First low transverse cesarean was for active herpes type 2 infections; vaginal culture at 39 weeks pregnancy was positive. b. First and second caesareans were for cephalopelvic disproportion. c. First caesarean through a classic incision as a result of severe fetal distress. d. First low transverse caesarean was for breech position. Fetus in this pregnancy is in a vertex presentation. 11. Nurse Ryan is aware that the best initial approach when trying to take a crying toddlers temperature is: a. Talk to the mother first and then to the

toddler. b. Bring extra help so it can be done quickly. c. Encourage the mother to hold the child. d. Ignore the crying and screaming. 12. Baby Tina a 3 month old infant just had a cleft lip and palate repair. What should the nurse do to prevent trauma to operative site? a. Avoid touching the suture line, even when cleaning. b. Place the baby in prone position. c. Give the baby a pacifier. d. Place the infants arms in soft elbow restraints. 13. Which action should nurse Marian include in the care plan for a 2 month old with heart failure? a. Feed the infant when he cries. b. Allow the infant to rest before feeding. c. Bathe the infant and administer medications before feeding. d. Weigh and bathe the infant before feeding. 14. Nurse Hazel is teaching a mother who plans to discontinue breast feeding after 5 months. The nurse should advise her to include which foods in her infants diet? a. Skim milk and baby food. b. Whole milk and baby food. c. Iron-rich formula only. d. Iron-rich formula and baby food. 15. Mommy Linda is playing with her infant, who is sitting securely alone on the floor of the clinic. The mother hides a toy behind her back and the infant looks for it. The nurse is aware that estimated age of the infant would be: a. 6 months

b. 4 months c. 8 months d. 10 months 16. Which of the following is the most prominent feature of public health nursing? a. It involves providing home care to sick people who are not confined in the hospital. b. Services are provided free of charge to people within the catchments area. c. The public health nurse functions as part of a team providing a public health nursing services. d. Public health nursing focuses on preventive, not curative, services. 17. When the nurse determines whether resources were maximized in implementing Ligtas Tigdas, she is evaluating a. Effectiveness b. Efficiency c. Adequacy d. Appropriateness 18. Vangie is a new B.S.N. graduate. She wants to become a Public Health Nurse. Where should she apply? a. Department of Health b. Provincial Health Office c. Regional Health Office d. Rural Health Unit 19. Tony is aware the Chairman of the Municipal Health Board is: a. Mayor b. Municipal Health Officer c. Public Health Nurse d. Any qualified physician

20. Myra is the public health nurse in a municipality with a total population of about 20,000. There are 3 rural health midwives among the RHU personnel. How many more midwife items will the RHU need? a. 1 b. 2 c. 3 d. The RHU does not need any more midwife item. 21. According to Freeman and Heinrich, community health nursing is a developmental service. Which of the following best illustrates this statement? a. The community health nurse continuously develops himself personally and professionally. b. Health education and community organizing are necessary in providing community health services. c. Community health nursing is intended primarily for health promotion and prevention and treatment of disease. d. The goal of community health nursing is to provide nursing services to people in their own places of residence. 22. Nurse Tina is aware that the disease declared through Presidential Proclamation No. 4 as a target for eradication in the Philippines is? a. Poliomyelitis b. Measles c. Rabies d. Neonatal tetanus 23. May knows that the step in community organizing that involves training of potential leaders in the community is:

a. Integration b. Community organization c. Community study d. Core group formation 24. Beth a public health nurse takes an active role in community participation. What is the primary goal of community organizing? a. To educate the people regarding community health problems b. To mobilize the people to resolve community health problems c. To maximize the communitys resources in dealing with health problems. d. To maximize the communitys resources in dealing with health problems. 25. Tertiary prevention is needed in which stage of the natural history of disease? a. Pre-pathogenesis b. Pathogenesis c. Prodromal d. Terminal 26. The nurse is caring for a primigravid client in the labor and delivery area. Which condition would place the client at risk for disseminated intravascular coagulation (DIC)? a. Intrauterine fetal death. b. Placenta accreta. c. Dysfunctional labor. d. Premature rupture of the membranes. 27. A fullterm client is in labor. Nurse Betty is aware that the fetal heart rate would be: a. 80 to 100 beats/minute b. 100 to 120 beats/minute

c. 120 to 160 beats/minute d. 160 to 180 beats/minute 28. The skin in the diaper area of a 7 month old infant is excoriated and red. Nurse Hazel should instruct the mother to: a. Change the diaper more often. b. Apply talc powder with diaper changes. c. Wash the area vigorously with each diaper change. d. Decrease the infants fluid intake to decrease saturating diapers. 29. Nurse Carla knows that the common cardiac anomalies in children with Down Syndrome (trisomy 21) is: a. Atrial septal defect b. Pulmonic stenosis c. Ventricular septal defect d. Endocardial cushion defect 30. Malou was diagnosed with severe preeclampsia is now receiving I.V. magnesium sulfate. The adverse effects associated with magnesium sulfate is: a. Anemia b. Decreased urine output c. Hyperreflexia d. Increased respiratory rate 31. A 23 year old client is having her menstrual period every 2 weeks that last for 1 week. This type of menstrual pattern is bets defined by: a. Menorrhagia b. Metrorrhagia c. Dyspareunia d. Amenorrhea 32. Jannah is admitted to the labor and delivery

unit. The critical laboratory result for this client would be: a. Oxygen saturation b. Iron binding capacity c. Blood typing d. Serum Calcium 33. Nurse Gina is aware that the most common condition found during the second-trimester of pregnancy is: a. Metabolic alkalosis b. Respiratory acidosis c. Mastitis d. Physiologic anemia 34. Nurse Lynette is working in the triage area of an emergency department. She sees that several pediatric clients arrive simultaneously. The client who needs to be treated first is: a. A crying 5 year old child with a laceration on his scalp. b. A 4 year old child with a barking coughs and flushed appearance. c. A 3 year old child with Down syndrome who is pale and asleep in his mothers arms. d. A 2 year old infant with stridorous breath sounds, sitting up in his mothers arms and drooling. 35. Maureen in her third trimester arrives at the emergency room with painless vaginal bleeding. Which of the following conditions is suspected? a. Placenta previa b. Abruptio placentae c. Premature labor d. Sexually transmitted disease

36. A young child named Richard is suspected of having pinworms. The community nurse collects a stool specimen to confirm the diagnosis. The nurse should schedule the collection of this specimen for: a. Just before bedtime b. After the child has been bathe c. Any time during the day d. Early in the morning 37. In doing a childs admission assessment, Nurse Betty should be alert to note which signs or symptoms of chronic lead poisoning? a. Irritability and seizures b. Dehydration and diarrhea c. Bradycardia and hypotension d. Petechiae and hematuria 38. To evaluate a womans understanding about the use of diaphragm for family planning, Nurse Trish asks her to explain how she will use the appliance. Which response indicates a need for further health teaching? a. I should check the diaphragm carefully for holes every time I use it b. I may need a different size of diaphragm if I gain or lose weight more than pounds c. The diaphragm must be left in place for atleast hours after intercourse d. I really need to use the diaphragm and jelly most during the middle of my menstrual cycle 39. Hypoxia is a common complication of laryngotracheobronchitis. Nurse Oliver should frequently assess a child with laryngotracheobronchitis for:

a. Drooling b. Muffled voice c. Restlessness d. Low-grade fever 40. How should Nurse Michelle guide a child who is blind to walk to the playroom? a. Without touching the child, talk continuously as the child walks down the hall. b. Walk one step ahead, with the childs hand on the nurses elbow c. Walk slightly behind, gently guiding the child forward. d. Walk next to the child, holding the childs hand 41. When assessing a newborn diagnosed with ductus arteriosus, Nurse Olivia should expect that the child most likely would have an: a. Loud, machinery-like murmur. b. Bluish color to the lips. c. Decreased BP reading in the upper extremities d. Increased BP reading in the upper extremities. 42. The reason nurse May keeps the neonate in a neutral thermal environment is that when a newborn becomes too cool, the neonate requires: a. Less oxygen, and the newborns metabolic rate increases. b. More oxygen, and the newborns metabolic rate decreases. c. More oxygen, and the newborns metabolic rate increases. d. Less oxygen, and the newborns

metabolic rate decreases. 43. Before adding potassium to an infants IVline, Nurse Ron must be sure to assess whether this infant has: a. Stable blood pressure b. Patant fontanelles c. Moros reflex d. Voided 44. Nurse Carla should know that the most common causative factor of dermatitis in infants and younger children is: a. Baby oil b. Baby lotion c. Laundry detergent d. Powder with cornstarch 45. During tube feeding, how far above an infants stomach should the nurse hold the syringe with formula? a. 6 inches b. 12 inches c. 18 inches d. 24 inches 46. In a mothersclass, Nurse Lhynnete discussed childhood diseases such as chicken pox. Which of the following statements about chicken pox is correct? a. The older one gets, the more susceptible he becomes to the complications of chicken pox. b. A single attack of chicken pox will

prevent future episodes, including conditions such as shingles. c. To prevent an outbreak in the community, quarantine may be imposed by health authorities. d. Chicken pox vaccine is best given when there is an impending outbreak in the community. 47. Barangay Pinoy had an outbreak of German measles. To prevent congenital rubella, what is the BEST advice that you can give to women in the first trimester of pregnancy in the barangay Pinoy? a. Advise them on the signs of German measles. b. Avoid crowded places, such as markets and movie houses. c. Consult at the health center where rubella vaccine may be given. d. Consult a physician who may give them rubella immunoglobulin. 48. Myrna a public health nurse knows that to determine possible sources of sexually transmitted infections, the BEST method that may be undertaken is: a. Contact tracing b. Community survey c. Mass screening tests d. Interview of suspects 49. A 33-year old female client came for consultation at the health center with the chief complaint of fever for a week. Accompanying symptoms were muscle pains and body malaise. A week after the start of fever, the client noted

yellowish discoloration of his sclera. History showed that he waded in flood waters about 2 weeks before the onset of symptoms. Based on her history, which disease condition will you suspect? a. Hepatitis A b. Hepatitis B c. Tetanus d. Leptospirosis 50. Mickey a 3-year old client was brought to the health center with the chief complaint of severe diarrhea and the passage of rice waterstools The client is most probably suffering from which condition? a. Giardiasis b. Cholera c. Amebiasis d. Dysentery 51. The most prevalent form of meningitis among children aged 2 months to 3 years is caused by which microorganism? a. Hemophilus influenzae b. Morbillivirus 79 c. Steptococcus pneumoniae d. Neisseria meningitidis 52. The student nurse is aware that the pathognomonic sign of measles is Kopliks spot and you may see Kopliks spot by inspecting the a. Nasal mucosa b. Buccal mucosa c. Skin on the abdomen d. Skin on neck 53. Angel was diagnosed as having Dengue fever.

You will say that there is slow capillary refill when the color of the nailbed that you pressed does not return within how many seconds? a. 3 seconds b. 6 seconds c. 9 seconds d. 10 seconds 54. In Integrated Management of Childhood Illness, the nurse is aware that the severe conditions generally require urgent referral to a hospital. Which of the following severe conditions DOES NOT always require urgent referral to a hospital? a. Mastoiditis b. Severe dehydration c. Severe pneumonia d. Severe febrile disease 55. Myrna a public health nurse will conduct outreach immunization in a barangay Masay with a population of about 1500. The estimated number of infants in the barangay would be: a. 45 infants b. 50 infants c. 55 infants d. 65 infants 56. The community nurse is aware that the biological used in Expanded Program on Immunization (EPI) should NOT be stored in the freezer? a. DPT b. Oral polio vaccine c. Measles vaccine d. MMR 57. It is the most effective way of controlling

schistosomiasis in an endemic area? a. Use of molluscicides b. Building of foot bridges c. Proper use of sanitary toilets d. Use of protective footwear, such as rubber boots 58. Several clients is newly admitted and diagnosed with leprosy. Which of the following clients should be classified as a case of multibacillary leprosy? a. 3 skin lesions, negative slit skin smear b. 3 skin lesions, positive slit skin smear c. 5 skin lesions, negative slit skin smear d. 5 skin lesions, positive slit skin smear 59. Nurses are aware that diagnosis of leprosy is highly dependent on recognition of symptoms. Which of the following is an early sign of leprosy? a. Macular lesions b. Inability to close eyelids c. Thickened painful nerves d. Sinking of the nosebridge 60. Marie brought her 10 month old infant for consultation because of fever, started 4 days prior to consultation. In determining malaria risk, what will you do? a. Perform a tourniquet test. b. Ask where the family resides. c. Get a specimen for blood smear. d. Ask if the fever is present every day. 61. Susie brought her 4 years old daughter to the RHU because of cough and colds. Following the IMCI assessment guide, which of the following is

a danger sign that indicates the need for urgent referral to a hospital? a. Inability to drink b. High grade fever c. Signs of severe dehydration d. Cough for more than 30 days 62. Jimmy a 2-year old child revealed baggy pants As a nurse, using the IMCI guidelines, how will you manage Jimmy? a. Refer the child urgently to a hospital for confinement. b. Coordinate with the social worker to enroll the child in a feeding program. c. Make a teaching plan for the mother, focusing on menu planning for her child. d. Assess and treat the child for health problems like infections and intestinal parasitism. 63. Gina is using Oresol in the management of diarrhea of her 3-year old child. She asked you what to do if her child vomits. As a nurse you will tell her to: a. Bring the child to the nearest hospital for further assessment. b. Bring the child to the health center for intravenous fluid therapy. c. Bring the child to the health center for assessment by the physician. d. Let the child rest for 10 minutes then continue giving Oresol more slowly. 64. Nikki a 5-month old infant was brought by his mother to the health center because of diarrhea for 4 to 5 times a day. Her skin goes back slowly after a skin pinch and her eyes are sunken. Using

the IMCI guidelines, you will classify this infant in which category? a. No signs of dehydration b. Some dehydration c. Severe dehydration d. The data is insufficient. 65. Chris a 4-month old infant was brought by her mother to the health center because of cough. His respiratory rate is 42/minute. Using the Integrated Management of Child Illness (IMCI) guidelines of assessment, his breathing is considered as: a. Fast b. Slow c. Normal d. Insignificant 66. Maylene had just received her 4th dose of tetanus toxoid. She is aware that her baby will have protection against tetanus for a. 1 year b. 3 years c. 5 years d. Lifetime 67. Nurse Ron is aware that unused BCG should be discarded after how many hours of reconstitution? a. 2 hours b. 4 hours c. 8 hours d. At the end of the day 68. The nurse explains to a breastfeeding mother that breast milk is sufficient for all of the babys nutrient needs only up to: a. 5 months

b. 6 months c. 1 year d. 2 years 69. Nurse Ron is aware that the gestational age of a conceptus that is considered viable (able to live outside the womb) is: a. 8 weeks b. 12 weeks c. 24 weeks d. 32 weeks 70. When teaching parents of a neonate the proper position for the neonates sleep, the nurse Patricia stresses the importance of placing the neonate on his back to reduce the risk of which of the following? a. Aspiration b. Sudden infant death syndrome (SIDS) c. Suffocation d. Gastroesophageal reflux (GER) 71. Which finding might be seen in baby James a neonate suspected of having an infection? a. Flushed cheeks b. Increased temperature c. Decreased temperature d. Increased activity level 72. Baby Jenny who is small-for-gestation is at increased risk during the transitional period for which complication? a. Anemia probably due to chronic fetal hyposia b. Hyperthermia due to decreased glycogen stores c. Hyperglycemia due to decreased glycogen stores

d. Polycythemia probably due to chronic fetal hypoxia 73. Marjorie has just given birth at weeks gestation. When the nurse assessing the neonate, which physical finding is expected? a. A sleepy, lethargic baby b. Lanugo covering the body c. Desquamation of the epidermis d. Vernix caseosa covering the body 74. After reviewing the Myrnas maternal history of magnesium sulfate during labor, which condition would nurse Richard anticipate as a potential problem in the neonate? a. Hypoglycemia b. Jitteriness c. Respiratory depression d. Tachycardia 75. Which symptom would indicate the Baby Alexandra was adapting appropriately to extrauterine life without difficulty? a. Nasal flaring b. Light audible grunting c. Respiratory rate 40 to 60 breaths/minute d. Respiratory rate 60 to 80 breaths/minute 76. When teaching umbilical cord care for Jennifer a new mother, the nurse Jenny would include which information? a. Apply peroxide to the cord with each diaper change b. Cover the cord with petroleum jelly after bathing c. Keep the cord dry and open to air d. Wash the cord with soap and water each day during a tub bath.

77. Nurse John is performing an assessment on a neonate. Which of the following findings is considered common in the healthy neonate? a. Simian crease b. Conjunctival hemorrhage c. Cystic hygroma d. Bulging fontanelle 78. Dr. Esteves decides to artificially rupture the membranes of a mother who is on labor. Following this procedure, the nurse Hazel checks the fetal heart tones for which the following reasons? a. To determine fetal well-being. b. To assess for prolapsed cord c. To assess fetal position d. To prepare for an imminent delivery. 79. Which of the following would be least likely to indicate anticipated bonding behaviors by new parents? a. The parentswillingness to touch and hold the new born. b. The parents expression of interest about the size of the new born. c. The parentsindication that they want to see the newborn. d. The parentsinteractions with each other. 80. Following a precipitous delivery, examination of the client's vagina reveals a fourth-degree laceration. Which of the following would be contraindicated when caring for this client? a. Applying cold to limit edema during the first 12 to 24 hours. b. Instructing the client to use two or more peripads to cushion the area.

c. Instructing the client on the use of sitz baths if ordered. d. Instructing the client about the importance of perineal (kegel) exercises. 81. A pregnant woman accompanied by her husband, seeks admission to the labor and delivery area. She states that she's in labor and says she attended the facility clinic for prenatal care. Which question should the nurse Oliver ask her first? a. Do you have any chronic illnesses? b. Do you have any allergies? c. What is your expected due date? d. Who will be with you during labor? 82. A neonate begins to gag and turns a dusky color. What should the nurse do first? a. Calm the neonate. b. Notify the physician. c. Provide oxygen via face mask as ordered d. Aspirate the neonates nose and mouth with a bulb syringe. 83. When a client states that her "water broke," which of the following actions would be inappropriate for the nurse to do? a. Observing the pooling of straw-colored fluid. b. Checking vaginal discharge with nitrazine paper. c. Conducting a bedside ultrasound for an amniotic fluid index. d. Observing for flakes of vernix in the vaginal discharge. 84. A baby girl is born 8 weeks premature. At birth, she has no spontaneous respirations but is successfully resuscitated. Within several hours

she develops respiratory grunting, cyanosis, tachypnea, nasal flaring, and retractions. She's diagnosed with respiratory distress syndrome, intubated, and placed on a ventilator. Which nursing action should be included in the baby's plan of care to prevent retinopathy of prematurity? a. Cover his eyes while receiving oxygen. b. Keep her body temperature low. c. Monitor partial pressure of oxygen (Pao2) levels. d. Humidify the oxygen. 85. Which of the following is normal newborn calorie intake? a. 110 to 130 calories per kg. b. 30 to 40 calories per lb of body weight. c. At least 2 ml per feeding d. 90 to 100 calories per kg 86. Nurse John is knowledgeable that usually individual twins will grow appropriately and at the same rate as singletons until how many weeks? a. 16 to 18 weeks b. 18 to 22 weeks c. 30 to 32 weeks d. 38 to 40 weeks 87. Which of the following classifications applies to monozygotic twins for whom the cleavage of the fertilized ovum occurs more than 13 days after fertilization? a. conjoined twins b. diamniotic dichorionic twins c. diamniotic monochorionic twin d. monoamniotic monochorionic twins

88. Tyra experienced painless vaginal bleeding has just been diagnosed as having a placenta previa. Which of the following procedures is usually performed to diagnose placenta previa? a. Amniocentesis b. Digital or speculum examination c. External fetal monitoring d. Ultrasound 89. Nurse Arnold knows that the following changes in respiratory functioning during pregnancy is considered normal: a. Increased tidal volume b. Increased expiratory volume c. Decreased inspiratory capacity d. Decreased oxygen consumption 90. Emily has gestational diabetes and it is usually managed by which of the following therapy? a. Diet b. Long-acting insulin c. Oral hypoglycemic d. Oral hypoglycemic drug and insulin 91. Magnesium sulfate is given to Jemma with preeclampsia to prevent which of the following condition? a. Hemorrhage b. Hypertension c. Hypomagnesemia d. Seizure 92. Cammile with sickle cell anemia has an increased risk for having a sickle cell crisis during pregnancy. Aggressive management of a sickle cell crisis includes which of the following measures? a. Antihypertensive agents

b. Diuretic agents c. I.V. fluids d. Acetaminophen (Tylenol) for pain 93. Which of the following drugs is the antidote for magnesium toxicity? a. Calcium gluconate (Kalcinate) b. Hydralazine (Apresoline) c. Naloxone (Narcan) d. Rho (D) immune globulin (RhoGAM) 94. Marlyn is screened for tuberculosis during her first prenatal visit. An intradermal injection of purified protein derivative (PPD) of the tuberculin bacilli is given. She is considered to have a positive test for which of the following results? a. An indurated wheal under 10 mm in diameter appears in 6 to 12 hours. b. An indurated wheal over 10 mm in diameter appears in 48 to 72 hours. c. A flat circumcised area under 10 mm in diameter appears in 6 to 12 hours. d. A flat circumcised area over 10 mm in diameter appears in 48 to 72 hours. 95. Dianne, 24 year-old is weekspregnant arrives at her physicians office with complaints of fever, nausea, vomiting, malaise, unilateral flank pain, and costovertebral angle tenderness. Which of the following diagnoses is most likely? a. Asymptomatic bacteriuria b. Bacterial vaginosis c. Pyelonephritis d. Urinary tract infection (UTI) 96. Rh isoimmunization in a pregnant client develops during which of the following conditions?

a. Rh-positive maternal blood crosses into fetal blood, stimulating fetal antibodies. b. Rh-positive fetal blood crosses into maternal blood, stimulating maternal antibodies. c. Rh-negative fetal blood crosses into maternal blood, stimulating maternal antibodies. d. Rh-negative maternal blood crosses into fetal blood, stimulating fetal antibodies. 97. To promote comfort during labor, the nurse John advises a client to assume certain positions and avoid others. Which position may cause maternal hypotension and fetal hypoxia? a. Lateral position b. Squatting position c. Supine position d. Standing position 98. Celeste who used heroin during her pregnancy delivers a neonate. When assessing the neonate, the nurse Lhynnette expects to find: a. Lethargy 2 days after birth. b. Irritability and poor sucking. c. A flattened nose, small eyes, and thin lips. d. Congenital defects such as limb anomalies. 99. The uterus returns to the pelvic cavity in which of the following time frames? a. 7th to 9th day postpartum. b. 2 weeks postpartum. c. End of 6th week postpartum. d. When the lochia changes to alba. 100. Maureen, a primigravida client, age 20, has just completed a difficult, forceps-assisted delivery of twins. Her labor was unusually

long and required oxytocin (Pitocin) augmentation. The nurse who's caring for her should stay alert for: a. Uterine inversion b. Uterine atony c. Uterine involution d. Uterine discomfort

TEST III - Care of Clients with Physiologic and Psychosocial Alterations 1. Nurse Michelle should know that the drainage is normal 4 days after a sigmoid colostomy when the stool is: a. Green liquid b. Solid formed c. Loose, bloody d. Semiformed 2. Where would nurse Kristine place the call light for a male client with a right-sided brain attack and left homonymous hemianopsia? a. On the clients right side b. On the clients left side c. Directly in front of the client d. Where the client like 3. A male client is admitted to the emergency department following an accident. What are the first nursing actions of the nurse? a. Check respiration, circulation, neurological response.

b. Align the spine, check pupils, and check for hemorrhage. c. Check respirations, stabilize spine, and check circulation. d. Assess level of consciousness and circulation. 4. In evaluating the effect of nitroglycerin, Nurse Arthur should know that it reduces preload and relieves angina by: a. Increasing contractility and slowing heart rate. b. Increasing AV conduction and heart rate. c. Decreasing contractility and oxygen consumption. d. Decreasing venous return through vasodilation. 5. Nurse Patricia finds a female client who is postmyocardial infarction (MI) slumped on the side rails of the bed and unresponsive to shaking or shouting. Which is the nurse next action? a. Call for help and note the time. b. Clear the airway c. Give two sharp thumps to the precordium, and check the pulse. d. Administer two quick blows. 6. Nurse Monett is caring for a client recovering from gastro-intestinal bleeding. The nurse should: a. Plan care so the client can receive 8 hours of uninterrupted sleep each night. b. Monitor vital signs every 2 hours. c. Make sure that the client takes food and medications at prescribed intervals.

d. Provide milk every 2 to 3 hours. 7. A male client was on warfarin (Coumadin) before admission, and has been receiving heparin I.V. for 2 days. The partial thromboplastin time (PTT) is 68 seconds. What should Nurse Carla do? a. Stop the I.V. infusion of heparin and notify the physician. b. Continue treatment as ordered. c. Expect the warfarin to increase the PTT. d. Increase the dosage, because the level is lower than normal. 8. A client undergone ileostomy, when should the drainage appliance be applied to the stoma? a. 24 hours later, when edema has subsided. b. In the operating room. c. After the ileostomy begin to function. d. When the client is able to begin self-care procedures. 9. A client undergone spinal anesthetic, it will be important that the nurse immediately position the client in: a. On the side, to prevent obstruction of airway by tongue. b. Flat on back. c. On the back, with knees flexed 15 degrees. d. Flat on the stomach, with the head turned to the side. 10. While monitoring a male client several hours after a motor vehicle accident, which assessment data suggest increasing intracranial pressure?

a. Blood pressure is decreased from 160/90 to 110/70. b. Pulse is increased from 87 to 95, with an occasional skipped beat. c. The client is oriented when aroused from sleep, and goes back to sleep immediately. d. The client refuses dinner because of anorexia. 11. Mrs. Cruz, 80 years old is diagnosed with pneumonia. Which of the following symptoms may appear first? a. Altered mental status and dehydration b. Fever and chills c. Hemoptysis and Dyspnea d. Pleuritic chest pain and cough 12. A male client has active tuberculosis (TB). Which of the following symptoms will be exhibit? a. Chest and lower back pain b. Chills, fever, night sweats, and hemoptysis c. Fever of more than 104F (40C) and nausea d. Headache and photophobia 13. Mark, a 7-year-old client is brought to the emergency departmentHes tachypneic and afebrile and has a respiratory rate of 36 breaths/minute and has a nonproductive cough. He recently had a cold. Form this history; the client may have which of the following conditions? a. Acute asthma b. Bronchial pneumonia

c. Chronic obstructive pulmonary disease (COPD) d. Emphysema 14. Marichu was given morphine sulfate for pain. She is sleeping and her respiratory rate is 4 breathsminuteIf action isnt taken quickly, she might have which of the following reactions? a. Asthma attack b. Respiratory arrest c. Seizure d. Wake up on his own 15. A 77-year-old male client is admitted for elective knee surgery. Physical examination reveals shallow respirations but no sign of respiratory distress. Which of the following is a normal physiologic change related to aging? a. Increased elastic recoil of the lungs b. Increased number of functional capillaries in the alveoli c. Decreased residual volume d. Decreased vital capacity 16. Nurse John is caring for a male client receiving lidocaine I.V. Which factor is the most relevant to administration of this medication? a. Decrease in arterial oxygen saturation (SaO2) when measured with a pulse oximeter. b. Increase in systemic blood pressure. c. Presence of premature ventricular contractions (PVCs) on a cardiac monitor. d. Increase in intracranial pressure (ICP).

17. Nurse Ron is caring for a male client taking an anticoagulant. The nurse should teach the client to: a. Report incidents of diarrhea. b. Avoid foods high in vitamin K c. Use a straight razor when shaving. d. Take aspirin to pain relief. 18. Nurse Lhynnette is preparing a site for the insertion of an I.V. catheter. The nurse should treat excess hair at the site by: a. Leaving the hair intact b. Shaving the area c. Clipping the hair in the area d. Removing the hair with a depilatory. 19. Nurse Michelle is caring for an elderly female with osteoporosis. When teaching the client, the nurse should include information about which major complication: a. Bone fracture b. Loss of estrogen c. Negative calcium balance d. Dowagers hump 20. Nurse Len is teaching a group of women to perform BSE. The nurse should explain that the purpose of performing the examination is to discover: a. Cancerous lumps b. Areas of thickness or fullness c. Changes from previous examinations. d. Fibrocystic masses 21. When caring for a female client who is being treated for hyperthyroidism, it is important to: a. Provide extra blankets and clothing to keep the client warm.

b. Monitor the client for signs of restlessness, sweating, and excessive 93 weight loss during thyroid replacement therapy. c. Balance the clients periods of activity and rest. d. Encourage the client to be active to prevent constipation. 22. Nurse Kris is teaching a client with history of atherosclerosis. To decrease the risk of atherosclerosis, the nurse should encourage the client to: a. Avoid focusing on his weight. b. Increase his activity level. c. Follow a regular diet. d. Continue leading a high-stress lifestyle. 23. Nurse Greta is working on a surgical floor. Nurse Greta must logroll a client following a: a. Laminectomy b. Thoracotomy c. Hemorrhoidectomy d. Cystectomy. 24. A 55-year old client underwent cataract removal with intraocular lens implant. Nurse Oliver is giving the client discharge instructions. These instructions should include which of the following? a. Avoid lifting objects weighing more than 5 lb (2.25 kg). b. Lie on your abdomen when in bed c. Keep rooms brightly lit.

d. Avoiding straining during bowel movement or bending at the waist. 25. George should be taught about testicular examinations during: a. when sexual activity starts b. After age 69 c. After age 40 d. Before age 20. 26. A male client undergone a colon resection. While turning him, wound dehiscence with evisceration occurs. Nurse Trish first response is to: a. Call the physician b. Place a saline-soaked sterile dressing on the wound. c. Take a blood pressure and pulse. d. Pull the dehiscence closed. 27. Nurse Audrey is caring for a client who has suffered a severe cerebrovascular accident. During routine assessment, the nurse notices Cheyne- Strokes respirations. Cheyne-strokes respirations are: a. A progressively deeper breaths followed by shallower breaths with apneic periods. b. Rapid, deep breathing with abrupt pauses between each breath. c. Rapid, deep breathing and irregular breathing without pauses. d. Shallow breathing with an increased respiratory rate. 28. Nurse Bea is assessing a male client with heart failure. The breath sounds commonly auscultated in clients with heart failure are:

a. Tracheal b. Fine crackles c. Coarse crackles d. Friction rubs 29. The nurse is caring for Kenneth experiencing an acute asthma attack. The client stops wheezing and breath sounds arent audibleThe reason for this change is that: a. The attack is over. b. The airways are so swollen that no air cannot get through. c. The swelling has decreased. d. Crackles have replaced wheezes. 30. Mike with epilepsy is having a seizure. During the active seizure phase, the nurse should: a. Place the client on his back remove dangerous objects, and insert a bite block. b. Place the client on his side, remove dangerous objects, and insert a bite block. c. Place the client o his back, remove dangerous objects, and hold down his arms. d. Place the client on his side, remove dangerous objects, and protect his head. 31. After insertion of a cheat tube for a pneumothorax, a client becomes hypotensive with neck vein distention, tracheal shift, absent breath sounds, and diaphoresis. Nurse Amanda suspects a tension pneumothorax has occurred. What cause of tension pneumothorax should the nurse check for? a. Infection of the lung. b. Kinked or obstructed chest tube c. Excessive water in the water-seal chamber d. Excessive chest tube drainage

32. Nurse Maureen is talking to a male client; the client begins choking on his lunchHes coughing forcefully. The nurse should: a. Stand him up and perform the abdominal thrust maneuver from behind. b. Lay him down, straddle him, and perform the abdominal thrust maneuver. c. Leave him to get assistance d. Stay with him but not intervene at this time. 33. Nurse Ron is taking a health history of an 84 year old client. Which information will be most useful to the nurse for planning care? a. General health for the last 10 years. b. Current health promotion activities. c. Family history of diseases. d. Marital status. 34. When performing oral care on a comatose client, Nurse Krina should: a. Apply lemon glycerin to the clients lips at least every 2 hours. b. Brush the teeth with client lying supine. c. Place the client in a side lying position, with the head of the bed lowered. d. Clean the clients mouth with hydrogen peroxide. 35. A 77-year-old male client is admitted with a diagnosis of dehydration and change in mental statusHes being hydrated with LVfluids When the nurse takes his vital signs, she notes he has a fever of 103F (39.4C) a cough producing yellow sputum and pleuritic chest pain. The nurse suspects this client may have which of the following conditions? a. Adult respiratory distress syndrome (ARDS)

b. Myocardial infarction (MI) c. Pneumonia d. Tuberculosis 36. Nurse Oliver is working in an outpatient clinic. He has been alerted that there is an outbreak of tuberculosis (TB). Which of the following clients entering the clinic today most likely to have TB? a. A 16-year-old female high school student b. A 33-year-old day-care worker c. A 43-yesr-old homeless man with a history of alcoholism d. A 54-year-old businessman 37. Virgie with a positive Mantoux test result will be sent for a chest X-ray. The nurse is aware that which of the following reasons this is done? a. To confirm the diagnosis b. To determine if a repeat skin test is needed c. To determine the extent of lesions d. To determine if this is a primary or secondary infection 38. Kennedy with acute asthma showing inspiratory and expiratory wheezes and a decreased forced expiratory volume should be treated with which of the following classes of medication right away? a. Beta-adrenergic blockers b. Bronchodilators c. Inhaled steroids d. Oral steroids 39. Mr. Vasquez 56-year-old client with a 40-year history of smoking one to two packs of cigarettes per day has a chronic cough producing thick sputum, peripheral edema and cyanotic nail

beds. Based on this information, he most likely has which of the following conditions? a. Adult respiratory distress syndrome (ARDS) b. Asthma c. Chronic obstructive bronchitis d. Emphysema Situation: Francis, age 46 is admitted to the hospital with diagnosis of Chronic Lymphocytic Leukemia. 40. The treatment for patients with leukemia is bone marrow transplantation. Which statement about bone marrow transplantation is not correct? a. The patient is under local anesthesia during the procedure b. The aspirated bone marrow is mixed with heparin. c. The aspiration site is the posterior or anterior iliac crest. 95 d. The recipient receives cyclophosphamide (Cytoxan) for 4 consecutive days before the procedure. 41. After several days of admission, Francis becomes disoriented and complains of frequent headaches. The nurse in-charge first action would be: a. Call the physician b. Document the patients status in his charts. c. Prepare oxygen treatment d. Raise the side rails 42. During routine care, Francis asks the nurse,

How can I be anemic if this disease causes increased my white blood cell production?The nurse in-charge best response would be that the increased number of white blood cells (WBC) is: a. Crowd red blood cells b. Are not responsible for the anemia. c. Uses nutrients from other cells d. Have an abnormally short life span of cells. 43. Diagnostic assessment of Francis would probably not reveal: a. Predominance of lymhoblasts b. Leukocytosis c. Abnormal blast cells in the bone marrow d. Elevated thrombocyte counts 44. Robert, a 57-year-old client with acute arterial occlusion of the left leg undergoes an emergency embolectomy. Six hours later, the nurse isnt able to obtain pulses in his left foot using Doppler ultrasound. The nurse immediately notifies the physician, and asks her to prepare the client for surgery. As the nurse enters the clients room to prepare him, he states that he wont have any more surgery. Which of the following is the best initial response by the nurse? a. Explain the risks of not having the surgery b. Notifying the physician immediately c. Notifying the nursing supervisor d. Recording the clients refusal in the nursesnotes 45. During the endorsement, which of the following clients should the on-duty nurse assess first?

a. The 58-year-old client who was admitted 2 days ago with heart failure, blood pressure of 126/76 mm Hg, and a respiratory rate of 22 breaths/ minute. b. The 89-year-old client with end-stage right-sided heart failure, blood pressure of mm Hg, and a do not resuscitateorder c. The 62-year-old client who was admitted 1 day ago with thrombophlebitis and is receiving L.V. heparin d. The 75-year-old client who was admitted 1 hour ago with new-onset atrial fibrillation and is receiving L.V. dilitiazem (Cardizem) 46. Honey, a 23-year old client complains of substernal chest pain and states that her heart feels like its racing out of the chestShe reports no history of cardiac disorders. The nurse attaches her to a cardiac monitor and notes sinus tachycardia with a rate of 136beats/minutes. Breath sounds are clear and the respiratory rate is 26 breaths/minutes. Which of the following drugs should the nurse question the client about using? a. Barbiturates b. Opioids c. Cocaine d. Benzodiazepines 47. A 51-year-old female client tells the nurse incharge that she has found a painless lump in her right breast during her monthly selfexamination. Which assessment finding would

strongly suggest that this client's lump is cancerous? a. Eversion of the right nipple and mobile mass b. Nonmobile mass with irregular edges c. Mobile mass that is soft and easily delineated d. Nonpalpable right axillary lymph nodes 48. A 35-year-old client with vaginal cancer asks the nurse, "What is the usual treatment for this type of cancer?" Which treatment should the nurse name? a. Surgery b. Chemotherapy c. Radiation d. Immunotherapy 49. Cristina undergoes a biopsy of a suspicious lesion. The biopsy report classifies the lesion according to the TNM staging system as follows: TIS, N0, M0. What does this classification mean? a. No evidence of primary tumor, no abnormal regional lymph nodes, and no evidence of distant metastasis b. Carcinoma in situ, no abnormal regional lymph nodes, and no evidence of distant metastasis c. Can't assess tumor or regional lymph nodes and no evidence of metastasis d. Carcinoma in situ, no demonstrable metastasis of the regional lymph nodes, and ascending degrees of distant metastasis 50. Lydia undergoes a laryngectomy to treat laryngeal cancer. When teaching the client how to care for the neck stoma, the nurse should include which instruction?

a. "Keep the stoma uncovered." b. "Keep the stoma dry." c. "Have a family member perform stoma care initially until you get used to the procedure." d. "Keep the stoma moist." 51. A 37-year-old client with uterine cancer asks the nurse, "Which is the most common type of cancer in women?" The nurse replies that it's breast cancer. Which type of cancer causes the most deaths in women? a. Breast cancer b. Lung cancer c. Brain cancer d. Colon and rectal cancer 52. Antonio with lung cancer develops Horner's syndrome when the tumor invades the ribs and affects the sympathetic nerve ganglia. When assessing for signs and symptoms of this syndrome, the nurse should note: a. miosis, partial eyelid ptosis, and anhidrosis on the affected side of the face. b. chest pain, dyspnea, cough, weight loss, and fever. c. arm and shoulder pain and atrophy of arm and hand muscles, both on the affected side. d. hoarseness and dysphagia. 53. Vic asks the nurse what PSA is. The nurse should reply that it stands for: a. prostate-specific antigen, which is used to screen for prostate cancer. b. protein serum antigen, which is used to determine protein levels.

c. pneumococcal strep antigen, which is a bacteria that causes pneumonia. d. Papanicolaou-specific antigen, which is used to screen for cervical cancer. 54. What is the most important postoperative instruction that nurse Kate must give a client who has just returned from the operating room after receiving a subarachnoid block? a. "Avoid drinking liquids until the gag reflex returns." b. "Avoid eating milk products for 24 hours." c. "Notify a nurse if you experience blood in your urine." d. "Remain supine for the time specified by the physician." 55. A male client suspected of having colorectal cancer will require which diagnostic study to confirm the diagnosis? a. Stool Hematest b. Carcinoembryonic antigen (CEA) c. Sigmoidoscopy d. Abdominal computed tomography (CT) scan 56. During a breast examination, which finding most strongly suggests that the Luz has breast cancer? a. Slight asymmetry of the breasts. b. A fixed nodular mass with dimpling of the overlying skin c. Bloody discharge from the nipple d. Multiple firm, round, freely movable masses that change with the menstrual cycle 57. A female client with cancer is being evaluated

for possible metastasis. Which of the following is one of the most common metastasis sites for cancer cells? a. Liver b. Colon c. Reproductive tract d. White blood cells (WBCs) 58. Nurse Mandy is preparing a client for magnetic resonance imaging (MRI) to confirm or rule out a spinal cord lesion. During the MRI scan, which of the following would pose a threat to the client? a. The client lies still. b. The client asks questions. c. The client hears thumping sounds. d. The client wears a watch and wedding band. 59. Nurse Cecile is teaching a female client about preventing osteoporosis. Which of the following teaching points is correct? a. Obtaining an X-ray of the bones every 3 years is recommended to detect bone loss. b. To avoid fractures, the client should avoid strenuous exercise. c. The recommended daily allowance of calcium may be found in a wide variety of foods. d. Obtaining the recommended daily allowance of calcium requires taking a calcium supplement. 60. Before Jacob undergoes arthroscopy, the nurse reviews the assessment findings for contraindications for this procedure. Which finding is a contraindication? a. Joint pain

b. Joint deformity c. Joint flexion of less than 50% d. Joint stiffness 61. Mr. Rodriguez is admitted with severe pain in the knees. Which form of arthritis is characterized by urate deposits and joint pain, usually in the feet and legs, and occurs primarily in men over age 30? a. Septic arthritis b. Traumatic arthritis c. Intermittent arthritis d. Gouty arthritis 62. A heparin infusion at 1,500 unit/hour is ordered for a 64-year-old client with stroke in evolution. The infusion contains 25,000 units of heparin in 500 ml of saline solution. How many milliliters per hour should be given? a. 15 ml/hour b. 30 ml/hour c. 45 ml/hour d. 50 ml/hour 63. A 76-year-old male client had a thromboembolic right stroke; his left arm is swollen. Which of the following conditions may cause swelling after a stroke? a. Elbow contracture secondary to spasticity b. Loss of muscle contraction decreasing venous return c. Deep vein thrombosis (DVT) due to immobility of the ipsilateral side d. Hypoalbuminemia due to protein

escaping from an inflamed glomerulus 64. Heberdens nodes are a common sign of osteoarthritis. Which of the following statement is correct about this deformity? a. It appears only in men b. It appears on the distal interphalangeal joint c. It appears on the proximal interphalangeal joint d. It appears on the dorsolateral aspect of the interphalangeal joint. 65. Which of the following statements explains the main difference between rheumatoid arthritis and osteoarthritis? a. Osteoarthritis is gender-specific, rheumatoid arthritis isnt b. Osteoarthritis is a localized disease rheumatoid arthritis is systemic c. Osteoarthritis is a systemic disease, rheumatoid arthritis is localized d. Osteoarthritis has dislocations and subluxations, rheumatoid arthritis doesnt 66. Mrs. Cruz uses a cane for assistance in walking. Which of the following statements is true about a cane or other assistive devices? a. A walker is a better choice than a cane. b. The cane should be used on the affected side c. The cane should be used on the unaffected side d. A client with osteoarthritis should be encouraged to ambulate without the cane

67. A male client with type 1 diabetes is scheduled to receive 30 U of 70/30 insulin. There is no 70/30 insulin available. As a substitution, the nurse may give the client: a. 9 U regular insulin and 21 U neutral protamine Hagedorn (NPH). b. 21 U regular insulin and 9 U NPH. c. 10 U regular insulin and 20 U NPH. d. 20 U regular insulin and 10 U NPH. 68. Nurse Len should expect to administer which medication to a client with gout? a. aspirin b. furosemide (Lasix) c. colchicines d. calcium gluconate (Kalcinate) 69. Mr. Domingo with a history of hypertension is diagnosed with primary hyperaldosteronism. This diagnosis indicates that the client's hypertension is caused by excessive hormone secretion from which of the following glands? a. Adrenal cortex b. Pancreas c. Adrenal medulla d. Parathyroid 70. For a diabetic male client with a foot ulcer, the doctor orders bed rest, a wet-to-dry dressing change every shift, and blood glucose monitoring before meals and bedtime. Why are wet-to-dry dressings used for this client? a. They contain exudate and provide a moist wound environment. b. They protect the wound from mechanical trauma and promote

healing. c. They debride the wound and promote healing by secondary intention. d. They prevent the entrance of microorganisms and minimize wound discomfort. 71. Nurse Zeny is caring for a client in acute addisonian crisis. Which laboratory data would the nurse expect to find? a. Hyperkalemia b. Reduced blood urea nitrogen (BUN) c. Hypernatremia d. Hyperglycemia 72. A client is admitted for treatment of the syndrome of inappropriate antidiuretic hormone (SIADH). Which nursing intervention is appropriate? a. Infusing I.V. fluids rapidly as ordered b. Encouraging increased oral intake c. Restricting fluids d. Administering glucose-containing I.V. fluids as ordered 73. A female client tells nurse Nikki that she has been working hard for the last 3 months to control her type 2 diabetes mellitus with diet and exercise. To determine the effectiveness of the client's efforts, the nurse should check: a. urine glucose level. b. fasting blood glucose level. c. serum fructosamine level. d. glycosylated hemoglobin level. 74. Nurse Trinity administered neutral protamine Hagedorn (NPH) insulin to a diabetic client at 7 a.m. At what time would the nurse expect the

client to be most at risk for a hypoglycemic reaction? a. 10:00 am b. Noon c. 4:00 pm d. 10:00 pm 75. The adrenal cortex is responsible for producing which substances? a. Glucocorticoids and androgens b. Catecholamines and epinephrine c. Mineralocorticoids and catecholamines d. Norepinephrine and epinephrine 76. On the third day after a partial thyroidectomy, Proserfina exhibits muscle twitching and hyperirritability of the nervous system. When questioned, the client reports numbness and tingling of the mouth and fingertips. Suspecting a life-threatening electrolyte disturbance, the nurse notifies the surgeon immediately. Which electrolyte disturbance most commonly follows thyroid surgery? a. Hypocalcemia b. Hyponatremia c. Hyperkalemia d. Hypermagnesemia 77. Which laboratory test value is elevated in clients who smoke and can't be used as a general indicator of cancer? a. Acid phosphatase level b. Serum calcitonin level c. Alkaline phosphatase level d. Carcinoembryonic antigen level

99 78. Francis with anemia has been admitted to the medical-surgical unit. Which assessment findings are characteristic of iron-deficiency anemia? a. Nights sweats, weight loss, and diarrhea b. Dyspnea, tachycardia, and pallor c. Nausea, vomiting, and anorexia d. Itching, rash, and jaundice 79. In teaching a female client who is HIV-positive about pregnancy, the nurse would know more teaching is necessary when the client says: a. The baby can get the virus from my placenta." b. "I'm planning on starting on birth control pills." c. "Not everyone who has the virus gives birth to a baby who has the virus." d. "I'll need to have a C-section if I become pregnant and have a baby." 80. When preparing Judy with acquired immunodeficiency syndrome (AIDS) for discharge to the home, the nurse should be sure to include which instruction? a. "Put on disposable gloves before bathing." b. "Sterilize all plates and utensils in boiling water." c. "Avoid sharing such articles as toothbrushes and razors." d. "Avoid eating foods from serving dishes shared by other family members." 81. Nurse Marie is caring for a 32-year-old client admitted with pernicious anemia. Which set of findings should the nurse expect when assessing

the client? a. Pallor, bradycardia, and reduced pulse pressure b. Pallor, tachycardia, and a sore tongue c. Sore tongue, dyspnea, and weight gain d. Angina, double vision, and anorexia 82. After receiving a dose of penicillin, a client develops dyspnea and hypotension. Nurse Celestina suspects the client is experiencing anaphylactic shock. What should the nurse do first? a. Page an anesthesiologist immediately and prepare to intubate the client. b. Administer epinephrine, as prescribed, and prepare to intubate the client if necessary. c. Administer the antidote for penicillin, as prescribed, and continue to monitor the client's vital signs. d. Insert an indwelling urinary catheter and begin to infuse I.V. fluids as ordered. 83. Mr. Marquez with rheumatoid arthritis is about to begin aspirin therapy to reduce inflammation. When teaching the client about aspirin, the nurse discusses adverse reactions to prolonged aspirin therapy. These include: a. weight gain. b. fine motor tremors. c. respiratory acidosis. d. bilateral hearing loss. 84. A 23-year-old client is diagnosed with human immunodeficiency virus (HIV). After recovering from the initial shock of the diagnosis, the client expresses a desire to learn as much as possible

about HIV and acquired immunodeficiency syndrome (AIDS). When teaching the client about the immune system, the nurse states that adaptive immunity is provided by which type of white blood cell? a. Neutrophil b. Basophil c. Monocyte d. Lymphocyte 85. In an individual with Sjgren's syndrome, nursing care should focus on: a. moisture replacement. b. electrolyte balance. c. nutritional supplementation. d. arrhythmia management. 86. During chemotherapy for lymphocytic leukemia, Mathew develops abdominal pain, fever, and "horse barn" smelling diarrhea. It would be most important for the nurse to advise the physician to order: a. enzyme-linked immunosuppressant assay (ELISA) test. b. electrolyte panel and hemogram. c. stool for Clostridium difficile test. d. flat plate X-ray of the abdomen. 87. A male client seeks medical evaluation for fatigue, night sweats, and a 20-lb weight loss in 6 weeks. To confirm that the client has been infected with the human immunodeficiency virus (HIV), the nurse expects the physician to order: 100 a. E-rosette immunofluorescence.

b. quantification of T-lymphocytes. c. enzyme-linked immunosorbent assay (ELISA). d. Western blot test with ELISA. 88. A complete blood count is commonly performed before a Joe goes into surgery. What does this test seek to identify? a. Potential hepatic dysfunction indicated by decreased blood urea nitrogen (BUN) and creatinine levels b. Low levels of urine constituents normally excreted in the urine c. Abnormally low hematocrit (HCT) and hemoglobin (Hb) levels d. Electrolyte imbalance that could affect the blood's ability to coagulate properly 89. While monitoring a client for the development of disseminated intravascular coagulation (DIC), the nurse should take note of what assessment parameters? a. Platelet count, prothrombin time, and partial thromboplastin time b. Platelet count, blood glucose levels, and white blood cell (WBC) count c. Thrombin time, calcium levels, and potassium levels d. Fibrinogen level, WBC, and platelet count 90. When taking a dietary history from a newly admitted female client, Nurse Len should remember that which of the following foods is a common allergen? a. Bread

b. Carrots c. Orange d. Strawberries 91. Nurse John is caring for clients in the outpatient clinic. Which of the following phone calls should the nurse return first? a. A client with hepatitis A who states, My arms and legs are itching b. A client with cast on the right leg who states, I have a funny feeling in my right leg c. A client with osteomyelitis of the spine who states, I am so nauseous that I cant eat d. A client with rheumatoid arthritis who states, I am having trouble sleeping 92. Nurse Sarah is caring for clients on the surgical floor and has just received report from the previous shift. Which of the following clients should the nurse see first? a. A 35-year-old admitted three hours ago with a gunshot wound; 1.5 cm area of dark drainage noted on the dressing. b. A 43-year-old who had a mastectomy two days ago; 23 ml of serosanguinous fluid noted in the Jackson-Pratt drain. c. A 59-year-old with a collapsed lung due to an accident; no drainage noted in the previous eight hours. d. A 62-year-old who had an abdominalperineal resection three days ago; client complaints of chills.

93. Nurse Eve is caring for a client who had a thyroidectomy 12 hours ago for treatment of Graves diseaseThe nurse would be most concerned if which of the following was observed? a. Blood pressure 138/82, respirations 16, oral temperature 99 degrees Fahrenheit. b. The client supports his head and neck when turning his head to the right. c. The client spontaneously flexes his wrist when the blood pressure is obtained. d. The client is drowsy and complains of sore throat. 94. Julius is admitted with complaints of severe pain in the lower right quadrant of the abdomen. To assist with pain relief, the nurse should take which of the following actions? a. Encourage the client to change positions frequently in bed. b. Administer Demerol 50 mg IM q 4 hours and PRN. c. Apply warmth to the abdomen with a heating pad. d. Use comfort measures and pillows to position the client. 95. Nurse Tina prepares a client for peritoneal dialysis. Which of the following actions should the nurse take first? a. Assess for a bruit and a thrill. b. Warm the dialysate solution. c. Position the client on the left side. 101 d. Insert a Foley catheter

96. Nurse Jannah teaches an elderly client with right-sided weakness how to use cane. Which of the following behaviors, if demonstrated by the client to the nurse, indicates that the teaching was effective? a. The client holds the cane with his right hand, moves the can forward followed by the right leg, and then moves the left leg. b. The client holds the cane with his right hand, moves the cane forward followed by his left leg, and then moves the right leg. c. The client holds the cane with his left hand, moves the cane forward followed by the right leg, and then moves the left leg. d. The client holds the cane with his left hand, moves the cane forward followed by his left leg, and then moves the right leg. 97. An elderly client is admitted to the nursing home setting. The client is occasionally confused and her gait is often unsteady. Which of the following actions, if taken by the nurse, is most appropriate? a. Ask the womans family to provide personal items such as photos or mementos. b. Select a room with a bed by the door so the woman can look down the hall. c. Suggest the woman eat her meals in the room with her roommate.

d. Encourage the woman to ambulate in the halls twice a day. 98. Nurse Evangeline teaches an elderly client how to use a standard aluminum walker. Which of the following behaviors, if demonstrated by the client, indicates that the nurses teaching was effective? a. The client slowly pushes the walker forward 12 inches, then takes small steps forward while leaning on the walker. b. The client lifts the walker, moves it forward 10 inches, and then takes several small steps forward. c. The client supports his weight on the walker while advancing it forward, then takes small steps while balancing on the walker. d. The client slides the walker 18 inches forward, then takes small steps while holding onto the walker for balance. 99. Nurse Deric is supervising a group of elderly clients in a residential home setting. The nurse knows that the elderly are at greater risk of developing sensory deprivation for what reason? a. Increased sensitivity to the side effects of medications. b. Decreased visual, auditory, and gustatory abilities. c. Isolation from their families and familiar surroundings. d. Decrease musculoskeletal function and mobility.

100. A male client with emphysema becomes restless and confused. What step should nurse Jasmine take next? a. Encourage the client to perform pursed lip breathing. b. Check the clients temperature c. Assess the clients potassium level d. Increase the clients oxygen flow rate

b. Polyuria c. Hypertension d. Shock 2. The immediate objective of nursing care for and overweight, mildly hypertensive male client with ureteral colic and hematuria is to decrease: a. Pain b. Weight c. Hematuria d. Hypertension 3. Matilda, with hyperthyroidism is to receive Lugols iodine solution before a subtotal thyroidectomy is performed. The nurse is aware that this medication is given to: a. Decrease the total basal metabolic rate. b. Maintain the function of the parathyroid glands. c. Block the formation of thyroxine by the thyroid gland. d. Decrease the size and vascularity of the thyroid gland. 4. Ricardo, was diagnosed with type I diabetes. The nurse is aware that acute hypoglycemia also can develop in the client who is diagnosed with: a. Liver disease b. Hypertension c. Type 2 diabetes

TEST IV - Care of Clients with Physiologic and Psychosocial Alterations 1. Randy has undergone kidney transplant, what assessment would prompt Nurse Katrina to suspect organ rejection? a. Sudden weight loss

d. Hyperthyroidism 5. Tracy is receiving combination chemotherapy for treatment of metastatic carcinoma. Nurse Ruby should monitor the client for the systemic side effect of:

a. Ascites b. Nystagmus c. Leukopenia d. Polycythemia 6. Norma, with recent colostomy expresses concern about the inability to control the passage of gas. Nurse Oliver should suggest that the client plan to: a. Eliminate foods high in cellulose. b. Decrease fluid intake at meal times. c. Avoid foods that in the past caused flatus. d. Adhere to a bland diet prior to social events. 7. Nurse Ron begins to teach a male client how to perform colostomy irrigations. The nurse would evaluate that the instructions were understood when the client states, I should a. Lie on my left side while instilling the irrigating solution b. Keep the irrigating container less than inches above the stoma c. Instill a minimum of 1200 ml of irrigating solution to stimulate evacuation of the bowel d. Insert the irrigating catheter deeper into the stoma if cramping occurs during the procedure 8. Patrick is in the oliguric phase of acute tubular necrosis and is experiencing fluid and electrolyte imbalances. The client is somewhat confused and complains of nausea and muscle weakness. As part of the prescribed therapy to correct this electrolyte imbalance, the nurse would expect

to: a. Administer Kayexalate b. Restrict foods high in protein c. Increase oral intake of cheese and milk. d. Administer large amounts of normal saline via I.V. 9. Mario has burn injury. After Forty48 hours, the physician orders for Mario 2 liters of IV fluid to be administered q12 h. The drop factor of the tubing is 10 gtt/ml. The nurse should set the flow to provide: a. 18 gtt/min b. 28 gtt/min c. 32 gtt/min d. 36 gtt/min 10. Terence suffered from burn injury. Using the rule of nines, which has the largest percent of burns? a. Face and neck b. Right upper arm and penis c. Right thigh and penis d. Upper trunk 11. Herbert, a 45 year old construction engineer is brought to the hospital unconscious after falling 112 from a 2-story building. When assessing the client, the nurse would be most concerned if the assessment revealed: a. Reactive pupils b. A depressed fontanel c. Bleeding from ears d. An elevated temperature 12. Nurse Sherry is teaching male client regarding his permanent artificial pacemaker. Which

information given by the nurse shows her knowledge deficit about the artificial cardiac pacemaker? a. take the pulse rate once a day, in the morning upon awakening b. May be allowed to use electrical appliances c. Have regular follow up care d. May engage in contact sports 13. The nurse is ware that the most relevant knowledge about oxygen administration to a male client with COPD is a. Oxygen at 1-2L/min is given to maintain the hypoxic stimulus for breathing. b. Hypoxia stimulates the central chemoreceptors in the medulla that makes the client breath. c. Oxygen is administered best using a nonrebreathing mask d. Blood gases are monitored using a pulse oximeter. 14. Tonny has undergoes a left thoracotomy and a partial pneumonectomy. Chest tubes are inserted, and one-bottle water-seal drainage is instituted in the operating room. In the postanesthesia care unit Tonny is placed in Fowler's position on either his right side or on his back. The nurse is aware that this position: a. Reduce incisional pain. b. Facilitate ventilation of the left lung. c. Equalize pressure in the pleural space. d. Increase venous return

15. Kristine is scheduled for a bronchoscopy. When teaching Kristine what to expect afterward, the nurse's highest priority of information would be: a. Food and fluids will be withheld for at least 2 hours. b. Warm saline gargles will be done q 2h. c. Coughing and deep-breathing exercises will be done q2h. d. Only ice chips and cold liquids will be allowed initially. 16. Nurse Tristan is caring for a male client in acute renal failure. The nurse should expect hypertonic, glucose, insulin infusions, and sodium bicarbonate to be used to treat: a. hypernatremia. b. hypokalemia. c. hyperkalemia. d. hypercalcemia. 17. Ms. X has just been diagnosed with condylomata acuminata (genital warts). What information is appropriate to tell this client? a. This condition puts her at a higher risk for cervical cancer; therefore, she should have a Papanicolaou (Pap) smear annually. b. The most common treatment is metronidazole (Flagyl), which should eradicate the problem within 7 to 10 days. c. The potential for transmission to her sexual partner will be eliminated if condoms are used every time they have sexual intercourse. d. The human papillomavirus (HPV), which

causes condylomata acuminata, can't be transmitted during oral sex. 18. Maritess was recently diagnosed with a genitourinary problem and is being examined in the emergency department. When palpating her kidneys, the nurse should keep which anatomical fact in mind? a. The left kidney usually is slightly higher than the right one. b. The kidneys are situated just above the adrenal glands. c. The average kidney is approximately 5 cm (2") long and 2 to 3 cm (" to 1-1/8") wide. d. The kidneys lie between the 10th and 12th thoracic vertebrae. 19. Jestoni with chronic renal failure (CRF) is admitted to the urology unit. The nurse is aware that the diagnostic test are consistent with CRF if the result is: a. Increased pH with decreased hydrogen ions. b. Increased serum levels of potassium, magnesium, and calcium. c. Blood urea nitrogen (BUN) 100 mg/dl and serum creatinine 6.5 mg/ dl. d. Uric acid analysis 3.5 mg/dl and phenolsulfonphthalein (PSP) excretion 75%. 20. Katrina has an abnormal result on a Papanicolaou test. After admitting that she read her chart while the nurse was out of the room, Katrina asks what dysplasia means. Which definition should the nurse provide? a. Presence of completely undifferentiated tumor cells that don't resemble cells of

the tissues of their origin. b. Increase in the number of normal cells in a normal arrangement in a tissue or an organ. c. Replacement of one type of fully differentiated cell by another in tissues where the second type normally isn't found. d. Alteration in the size, shape, and organization of differentiated cells. 21. During a routine checkup, Nurse Mariane assesses a male client with acquired immunodeficiency syndrome (AIDS) for signs and symptoms of cancer. What is the most common AIDS-related cancer? a. Squamous cell carcinoma b. Multiple myeloma c. Leukemia d. Kaposi's sarcoma 22. Ricardo is scheduled for a prostatectomy, and the anesthesiologist plans to use a spinal (subarachnoid) block during surgery. In the operating room, the nurse positions the client according to the anesthesiologist's instructions. Why does the client require special positioning for this type of anesthesia? a. To prevent confusion b. To prevent seizures c. To prevent cerebrospinal fluid (CSF) leakage d. To prevent cardiac arrhythmias 23. A male client had a nephrectomy 2 days ago and is now complaining of abdominal pressure and nausea. The first nursing action should be to: a. Auscultate bowel sounds. b. Palpate the abdomen. c. Change the client's position.

d. Insert a rectal tube. 24. Wilfredo with a recent history of rectal bleeding is being prepared for a colonoscopy. How should the nurse Patricia position the client for this test initially? a. Lying on the right side with legs straight b. Lying on the left side with knees bent c. Prone with the torso elevated d. Bent over with hands touching the floor 25. A male client with inflammatory bowel disease undergoes an ileostomy. On the first day after surgery, Nurse Oliver notes that the client's stoma appears dusky. How should the nurse interpret this finding? a. Blood supply to the stoma has been interrupted. b. This is a normal finding 1 day after surgery. c. The ostomy bag should be adjusted. d. An intestinal obstruction has occurred. 26. Anthony suffers burns on the legs, which nursing intervention helps prevent contractures? a. Applying knee splints b. Elevating the foot of the bed c. Hyperextending the client's palms d. Performing shoulder range-of-motion exercises 27. Nurse Ron is assessing a client admitted with second- and third-degree burns on the face, arms, and chest. Which finding indicates a potential problem? a. Partial pressure of arterial oxygen (PaO2) value of 80 mm Hg.

b. Urine output of 20 ml/hour. c. White pulmonary secretions. d. Rectal temperature of 100.6 F (38 C). 28. Mr. Mendoza who has suffered a cerebrovascular accident (CVA) is too weak to move on his own. To help the client avoid pressure ulcers, Nurse Celia should: a. Turn him frequently. b. Perform passive range-of-motion (ROM) exercises. c. Reduce the client's fluid intake. d. Encourage the client to use a footboard. 29. Nurse Maria plans to administer dexamethasone cream to a female client who has dermatitis over the anterior chest. How should the nurse apply this topical agent? a. With a circular motion, to enhance absorption. b. With an upward motion, to increase blood supply to the affected area c. In long, even, outward, and downward strokes in the direction of hair growth d. In long, even, outward, and upward strokes in the direction opposite hair growth 30. Nurse Kate is aware that one of the following classes of medication protects the ischemic myocardium by blocking catecholamines and sympathetic nerve stimulation is: a. Beta -adrenergic blockers b. Calcium channel blocker c. Narcotics d. Nitrates

31. A male client has jugular distention. On what position should the nurse place the head of the bed to obtain the most accurate reading of jugular vein distention? a. High Fowlers b. Raised 10 degrees c. Raised 30 degrees d. Supine position 32. The nurse is aware that one of the following classes of medications maximizes cardiac performance in clients with heart failure by increasing ventricular contractility? a. Beta-adrenergic blockers b. Calcium channel blocker c. Diuretics d. Inotropic agents 33. A male client has a reduced serum highdensity lipoprotein (HDL) level and an elevated lowdensity lipoprotein (LDL) level. Which of the following dietary modifications is not appropriate for this client? a. Fiber intake of 25 to 30 g daily b. Less than 30% of calories from fat c. Cholesterol intake of less than 300 mg daily d. Less than 10% of calories from saturated fat 34. A 37-year-old male client was admitted to the coronary care unit (CCU) 2 days ago with an acute myocardial infarction. Which of the following actions would breach the client confidentiality? a. The CCU nurse gives a verbal report to the nurse on the telemetry unit before transferring the client to that unit

b. The CCU nurse notifies the on-call physician about a change in the clients condition c. The emergency department nurse calls up the latest electrocardiogram results to check the clients progress d. At the clients request, the CCU nurse updates the clients wife on his condition 35. A male client arriving in the emergency department is receiving cardiopulmonary resuscitation from paramedics who are giving ventilations through an endotracheal (ET) tube that they placed in the clients homeDuring a pause in compressions, the cardiac monitor shows narrow QRS complexes and a heart rate of beats/minute with a palpable pulse. Which of the following actions should the nurse take first? a. Start an L.V. line and administer amiodarone (Cardarone), 300 mg L.V. over 10 minutes. b. Check endotracheal tube placement. c. Obtain an arterial blood gas (ABG) sample. d. Administer atropine, 1 mg L.V. 36. After cardiac surgery, a clients blood pressure measures 126/80 mm Hg. Nurse Katrina determines that mean arterial pressure (MAP) is which of the following? a. 46 mm Hg b. 80 mm Hg c. 95 mm Hg d. 90 mm Hg 37. A female client arrives at the emergency

department with chest and stomach pain and a report of black tarry stool for several months. Which of the following order should the nurse Oliver anticipate? a. Cardiac monitor, oxygen, creatine kinase and lactate dehydrogenase levels b. Prothrombin time, partial thromboplastin time, fibrinogen and fibrin split product values. c. Electrocardiogram, complete blood count, testing for occult blood, comprehensive serum metabolic panel. d. Electroencephalogram, alkaline phosphatase and aspartate aminotransferase levels, basic serum metabolic panel 38. Macario had coronary artery bypass graft (CABG) surgery 3 days ago. Which of the following conditions is suspected by the nurse when a decrease in platelet count from 230,000 ul to 5,000 ul is noted? a. Pancytopenia b. Idiopathic thrombocytopemic purpura (ITP) c. Disseminated intravascular coagulation (DIC) d. Heparin-associated thrombosis and thrombocytopenia (HATT) 39. Which of the following drugs would be ordered by the physician to improve the platelet count in a male client with idiopathic thrombocytopenic purpura (ITP)? a. Acetylsalicylic acid (ASA)

b. Corticosteroids c. Methotrezate d. Vitamin K 40. A female client is scheduled to receive a heart valve replacement with a porcine valve. Which of the following types of transplant is this? a. Allogeneic b. Autologous c. Syngeneic d. Xenogeneic 41. Marco falls off his bicycle and injuries his ankle. Which of the following actions shows the initial response to the injury in the extrinsic pathway? a. Release of Calcium b. Release of tissue thromboplastin c. Conversion of factors XII to factor XIIa d. Conversion of factor VIII to factor VIIIa 42. Instructions for a client with systemic lupus erythematosus (SLE) would include information about which of the following blood dyscrasias? a. Dresslers syndrome b. Polycythemia c. Essential thrombocytopenia d. Von Willebrands disease 43. The nurse is aware that the following symptom is most commonly an early indication of stage 1 Hodgkins disease? a. Pericarditis b. Night sweat c. Splenomegaly d. Persistent hypothermia 44. Francis with leukemia has neutropenia. Which of

the following functions must frequently assessed? a. Blood pressure b. Bowel sounds c. Heart sounds d. Breath sounds 45. The nurse knows that neurologic complications of multiple myeloma (MM) usually involve which of the following body system? a. Brain b. Muscle spasm c. Renal dysfunction d. Myocardial irritability 46. Nurse Patricia is aware that the average length of time from human immunodeficiency virus (HIV) infection to the development of acquired immunodeficiency syndrome (AIDS)? a. Less than 5 years b. 5 to 7 years c. 10 years d. More than 10 years 47. An 18-year-old male client admitted with heat stroke begins to show signs of disseminated intravascular coagulation (DIC). Which of the following laboratory findings is most consistent with DIC? a. Low platelet count b. Elevated fibrinogen levels c. Low levels of fibrin degradation products d. Reduced prothrombin time 48. Mario comes to the clinic complaining of fever, drenching night sweats, and unexplained weight

loss over the past 3 months. Physical examination reveals a single enlarged supraclavicular lymph node. Which of the following is the most probable diagnosis? a. Influenza b. Sickle cell anemia c. Leukemia d. Hodgkins disease 116 49. A male client with a gunshot wound requires an emergency blood transfusion. His blood type is AB negative. Which blood type would be the safest for him to receive? a. AB Rh-positive b. A Rh-positive c. A Rh-negative d. O Rh-positive Situation: Stacy is diagnosed with acute lymphoid leukemia (ALL) and beginning chemotherapy. 50. Stacy is discharged from the hospital following her chemotherapy treatments. Which statement of Stacys mother indicated that she understands when she will contact the physician? a. I should contact the physician if Stacy has difficulty in sleeping b. I will call my doctor if Stacy has persistent vomiting and diarrhea c. My physician should be called if Stacy is irritable and unhappy d. Should Stacy have continued hair loss, I need to call the doctor 51. Stacys mother states to the nurse that it is hard

to see Stacy with no hair. The best response for the nurse is: a. Stacy looks very nice wearing a hat b. You should not worry about her hair, just be glad that she is alive c. Yes it is upsettingBut try to cover up your feelings when you are with her or else she may be upset d. This is only temporaryStacy will regrow new hair in 3-6 months, but may be different in texture 52. Stacy has beginning stomatitis. To promote oral hygiene and comfort, the nurse in-charge should: a. Provide frequent mouthwash with normal saline. b. Apply viscous Lidocaine to oral ulcers as needed. c. Use lemon glycerine swabs every 2 hours. d. Rinse mouth with Hydrogen Peroxide. 53. During the administration of chemotherapy agents, Nurse Oliver observed that the IV site is red and swollen, when the IV is touched Stacy shouts in pain. The first nursing action to take is: a. Notify the physician b. Flush the IV line with saline solution c. Immediately discontinue the infusion d. Apply an ice pack to the site, followed by warm compress. 54. The term blue bloaterrefers to a male client which of the following conditions? a. Adult respiratory distress syndrome

(ARDS) b. Asthma c. Chronic obstructive bronchitis d. Emphysema 55. The term pink pufferrefers to the female client with which of the following conditions? a. Adult respiratory distress syndrome (ARDS) b. Asthma c. Chronic obstructive bronchitis d. Emphysema 56. Jose is in danger of respiratory arrest following the administration of a narcotic analgesic. An arterial blood gas value is obtained. Nurse Oliver would expect the paco2 to be which of the following values? a. 15 mm Hg b. 30 mm Hg c. 40 mm Hg d. 80 mm Hg 57. Timothys arterial blood gas ABGresults are as follows; pH 7.16; Paco2 80 mm Hg; Pao2 46 mm Hg; HCO3- 24mEq/L; Sao2 81%. This ABG result represents which of the following conditions? a. Metabolic acidosis b. Metabolic alkalosis c. Respiratory acidosis d. Respiratory alkalosis 58. Norma has started a new drug for hypertension. Thirty minutes after she takes the drug, she develops chest tightness and becomes short of breath and tachypneic. She has a decreased level

of consciousness. These signs indicate which of the following conditions? a. Asthma attack b. Pulmonary embolism c. Respiratory failure d. Rheumatoid arthritis 117 Situation: Mr. Gonzales was admitted to the hospital with ascites and jaundice. To rule out cirrhosis of the liver: 59. Which laboratory test indicates liver cirrhosis? a. Decreased red blood cell count b. Decreased serum acid phosphate level c. Elevated white blood cell count d. Elevated serum aminotransferase 60. 60.The biopsy of Mr. Gonzales confirms the diagnosis of cirrhosis. Mr. Gonzales is at increased risk for excessive bleeding primarily because of: a. Impaired clotting mechanism b. Varix formation c. Inadequate nutrition d. Trauma of invasive procedure 61. Mr. Gonzales develops hepatic encephalopathy. Which clinical manifestation is most common with this condition? a. Increased urine output b. Altered level of consciousness c. Decreased tendon reflex d. Hypotension 62. When Mr. Gonzales regained consciousness, the physician orders 50 ml of Lactose p.o. every 2

hours. Mr. Gozales develops diarrhea. The nurse best action would be: a. Ill see if your physician is in the hospital b. Maybe youre reacting to the drugI will withhold the next dose c. Ill lower the dosage as ordered so the drug causes only to stools a day d. Frequently, bowel movements are needed to reduce sodium level 63. Which of the following groups of symptoms indicates a ruptured abdominal aortic aneurysm? a. Lower back pain, increased blood pressure, decreased red blood cell (RBC) count, increased white blood (WBC) count. b. Severe lower back pain, decreased blood pressure, decreased RBC count, increased WBC count. c. Severe lower back pain, decreased blood pressure, decreased RBC count, decreased RBC count, decreased WBC count. d. Intermitted lower back pain, decreased blood pressure, decreased RBC count, increased WBC count. 64. After undergoing a cardiac catheterization, Tracy has a large puddle of blood under his buttocks. Which of the following steps should the nurse take first? a. Call for help. b. Obtain vital signs c. Ask the client to lift up

d. Apply gloves and assess the groin site 65. Which of the following treatment is a suitable surgical intervention for a client with unstable angina? a. Cardiac catheterization b. Echocardiogram c. Nitroglycerin d. Percutaneous transluminal coronary angioplasty (PTCA) 66. The nurse is aware that the following terms used to describe reduced cardiac output and perfusion impairment due to ineffective pumping of the heart is: a. Anaphylactic shock b. Cardiogenic shock c. Distributive shock d. Myocardial infarction (MI) 67. A client with hypertension asks the nurse which factors can cause blood pressure to drop to normal levels? a. Kidneysexcretion to sodium only b. Kidneysretention of sodium and water c. Kidneysexcretion of sodium and water d. Kidneysretention of sodium and excretion of water 68. Nurse Rose is aware that the statement that best explains why furosemide (Lasix) is administered to treat hypertension is: a. It dilates peripheral blood vessels. b. It decreases sympathetic cardioacceleration. c. It inhibits the angiotensin-coverting enzymes

d. It inhibits reabsorption of sodium and water in the loop of Henle. 69. Nurse Nikki knows that laboratory results supports the diagnosis of systemic lupus erythematosus (SLE) is: a. Elavated serum complement level b. Thrombocytosis, elevated sedimentation rate c. Pancytopenia, elevated antinuclear antibody (ANA) titer d. Leukocysis, elevated blood urea nitrogen (BUN) and creatinine levels 70. Arnold, a 19-year-old client with a mild concussion is discharged from the emergency department. Before discharge, he complains of a headache. When offered acetaminophen, his mother tells the nurse the headache is severe and she would like her son to have something stronger. Which of the following responses by the nurse is appropriate? a. Your son had a mild concussion, acetaminophen is strong enough b. Aspirin is avoided because of the danger of Reyes syndrome in children or young adults c. Narcotics are avoided after a head injury because they may hide a worsening condition d. Stronger medications may lead to vomiting, which increases the intracarnial pressure ICP 71. When evaluating an arterial blood gas from a male client with a subdural hematoma, the nurse notes the Paco2 is 30 mm Hg. Which of the following responses best describes the

result? a. Appropriate; lowering carbon dioxide (CO2) reduces intracranial pressure (ICP) b. Emergent; the client is poorly oxygenated c. Normal d. Significant; the client has alveolar hypoventilation 72. When prioritizing care, which of the following clients should the nurse Olivia assess first? a. A 17-year-old clients 24-hours postappendectomy b. A 33-year-old client with a recent diagnosis of Guillain-Barre syndrome c. A 50-year-old client 3 days postmyocardial infarction d. A 50-year-old client with diverticulitis 73. JP has been diagnosed with gout and wants to know why colchicine is used in the treatment of gout. Which of the following actions of colchicines explains why its effective for gout? a. Replaces estrogen b. Decreases infection c. Decreases inflammation d. Decreases bone demineralization 74. Norma asks for information about osteoarthritis. Which of the following statements about osteoarthritis is correct? a. Osteoarthritis is rarely debilitating b. Osteoarthritis is a rare form of arthritis c. Osteoarthritis is the most common form of arthritis d. Osteoarthritis afflicts people over 60 75. Ruby is receiving thyroid replacement therapy

develops the flu and forgets to take her thyroid replacement medicine. The nurse understands that skipping this medication will put the client at risk for developing which of the following lifethreatening complications? a. Exophthalmos b. Thyroid storm c. Myxedema coma d. Tibial myxedema 76. Nurse Sugar is assessing a client with Cushing's syndrome. Which observation should the nurse report to the physician immediately? a. Pitting edema of the legs b. An irregular apical pulse c. Dry mucous membranes d. Frequent urination 77. Cyrill with severe head trauma sustained in a car accident is admitted to the intensive care unit. Thirty-six hours later, the client's urine output suddenly rises above 200 ml/hour, leading the nurse to suspect diabetes insipidus. Which laboratory findings support the nurse's suspicion of diabetes insipidus? a. Above-normal urine and serum osmolality levels b. Below-normal urine and serum osmolality levels c. Above-normal urine osmolality level, below-normal serum osmolality level d. Below-normal urine osmolality level, above-normal serum osmolality level 78. Jomari is diagnosed with hyperosmolar hyperglycemic nonketotic syndrome (HHNS) is stabilized and prepared for discharge. When

preparing the client for discharge and home management, which of the following statements indicates that the client understands her condition and how to control it? a. "I can avoid getting sick by not becoming dehydrated and by paying attention to my need to urinate, drink, or eat more than usual." b. "If I experience trembling, weakness, and headache, I should drink a glass of soda that contains sugar." c. "I will have to monitor my blood glucose level closely and notify the physician if it's constantly elevated." d. "If I begin to feel especially hungry and thirsty, I'll eat a snack high in carbohydrates." 79. A 66-year-old client has been complaining of sleeping more, increased urination, anorexia, weakness, irritability, depression, and bone pain that interferes with her going outdoors. Based on these assessment findings, the nurse would suspect which of the following disorders? a. Diabetes mellitus b. Diabetes insipidus c. Hypoparathyroidism d. Hyperparathyroidism 80. Nurse Lourdes is teaching a client recovering from addisonian crisis about the need to take fludrocortisone acetate and hydrocortisone at home. Which statement by the client indicates an understanding of the instructions? a. "I'll take my hydrocortisone in the late afternoon, before dinner." b. "I'll take all of my hydrocortisone in the

morning, right after I wake up." c. "I'll take two-thirds of the dose when I wake up and one-third in the late afternoon." d. "I'll take the entire dose at bedtime." 81. Which of the following laboratory test results would suggest to the nurse Len that a client has a corticotropin-secreting pituitary adenoma? a. High corticotropin and low cortisol levels b. Low corticotropin and high cortisol levels c. High corticotropin and high cortisol levels d. Low corticotropin and low cortisol levels 82. A male client is scheduled for a transsphenoidal hypophysectomy to remove a pituitary tumor. Preoperatively, the nurse should assess for potential complications by doing which of the following? a. Testing for ketones in the urine b. Testing urine specific gravity c. Checking temperature every 4 hours d. Performing capillary glucose testing every 4 hours 83. Capillary glucose monitoring is being performed every 4 hours for a client diagnosed with diabetic ketoacidosis. Insulin is administered using a scale of regular insulin according to glucose results. At 2 p.m., the client has a capillary glucose level of 250 mg/dl for which he receives 8 U of regular insulin. Nurse Mariner should expect the dose's: a. onset to be at 2 p.m. and its peak to be at 3 p.m. b. onset to be at 2:15 p.m. and its peak to be at 3 p.m.

c. onset to be at 2:30 p.m. and its peak to be at 4 p.m. d. onset to be at 4 p.m. and its peak to be at 6 p.m. 84. The physician orders laboratory tests to confirm hyperthyroidism in a female client with classic signs and symptoms of this disorder. Which test result would confirm the diagnosis? a. No increase in the thyroid-stimulating hormone (TSH) level after 30 minutes during the TSH stimulation test b. A decreased TSH level c. An increase in the TSH level after 30 minutes during the TSH stimulation test d. Below-normal levels of serum triiodothyronine (T3) and serum thyroxine (T4) as detected by radioimmunoassay 85. Rico with diabetes mellitus must learn how to self-administer insulin. The physician has prescribed 10 U of U-100 regular insulin and 35 U of U-100 isophane insulin suspension (NPH) to be taken before breakfast. When teaching the client how to select and rotate insulin injection sites, the nurse should provide which instruction? a. "Inject insulin into healthy tissue with large blood vessels and nerves." b. "Rotate injection sites within the same anatomic region, not among different regions." c. "Administer insulin into areas of scar tissue or hypotrophy whenever possible."

d. "Administer insulin into sites above muscles that you plan to exercise heavily later that day." 86. Nurse Sarah expects to note an elevated serum glucose level in a client with hyperosmolar hyperglycemic nonketotic syndrome (HHNS). Which other laboratory finding should the nurse anticipate? a. Elevated serum acetone level b. Serum ketone bodies c. Serum alkalosis d. Below-normal serum potassium level 87. For a client with Graves' disease, which nursing intervention promotes comfort? a. Restricting intake of oral fluids b. Placing extra blankets on the client's bed c. Limiting intake of high-carbohydrate foods d. Maintaining room temperature in the low-normal range 88. Patrick is treated in the emergency department for a Colles' fracture sustained during a fall. What is a Colles' fracture? a. Fracture of the distal radius b. Fracture of the olecranon c. Fracture of the humerus d. Fracture of the carpal scaphoid 89. Cleo is diagnosed with osteoporosis. Which electrolytes are involved in the development of this disorder? a. Calcium and sodium b. Calcium and phosphorous c. Phosphorous and potassium d. Potassium and sodium 90. Johnny a firefighter was involved in

extinguishing a house fire and is being treated to smoke inhalation. He develops severe hypoxia 48 hours after the incident, requiring intubation and mechanical ventilation. He most likely has developed which of the following conditions? a. Adult respiratory distress syndrome (ARDS) b. Atelectasis c. Bronchitis d. Pneumonia 91. A 67-year-old client develops acute shortness of breath and progressive hypoxia requiring right femur. The hypoxia was probably caused by which of the following conditions? a. Asthma attack b. Atelectasis c. Bronchitis d. Fat embolism 92. A client with shortness of breath has decreased to absent breath sounds on the right side, from the apex to the base. Which of the following conditions would best explain this? a. Acute asthma b. Chronic bronchitis c. Pneumonia d. Spontaneous pneumothorax 93. A 62-year-old male client was in a motor vehicle accident as an unrestrained driverHes now in the emergency department complaining of difficulty of breathing and chest pain. On auscultation of his lung field, no breath sounds are present in the upper lobe. This client may have which of the following conditions? a. Bronchitis b. Pneumonia

c. Pneumothorax d. Tuberculosis (TB) 94. If a client requires a pneumonectomy, what fills the area of the thoracic cavity? a. The space remains filled with air only b. The surgeon fills the space with a gel c. Serous fluids fills the space and consolidates the region d. The tissue from the other lung grows over to the other side 95. Hemoptysis may be present in the client with a pulmonary embolism because of which of the following reasons? a. Alveolar damage in the infracted area b. Involvement of major blood vessels in the occluded area c. Loss of lung parenchyma d. Loss of lung tissue 96. Aldo with a massive pulmonary embolism will have an arterial blood gas analysis performed to determine the extent of hypoxia. The acid-base disorder that may be present is? a. Metabolic acidosis b. Metabolic alkalosis c. Respiratory acidosis d. Respiratory alkalosis 97. After a motor vehicle accident, Armand an 22year-old client is admitted with a pneumothorax. The surgeon inserts a chest tube and attaches it to a chest drainage system. Bubbling soon appears in the water seal chamber. Which of the following is the most likely cause of the bubbling? a. Air leak b. Adequate suction

c. Inadequate suction d. Kinked chest tube 98. Nurse Michelle calculates the IV flow rate for a postoperative client. The client receives 3,000 ml of Ringers lactate solution IV to run over hours. The IV infusion set has a drop factor of 10 drops per milliliter. The nurse should regulate the clients IV to deliver how many drops per minute? a. 18 b. 21 c. 35 d. 40 99. Mickey, a 6-year-old child with a congenital heart disorder is admitted with congestive heart failure. Digoxin (lanoxin) 0.12 mg is ordered for the child. The bottle of Lanoxin contains .05 mg of Lanoxin in 1 ml of solution. What amount should the nurse administer to the child? a. 1.2 ml b. 2.4 ml c. 3.5 ml d. 4.2 ml 100. Nurse Alexandra teaches a client about elastic stockings. Which of the following statements, if made by the client, indicates to the nurse that the teaching was successful? a. I will wear the stockings until the physician tells me to remove them b. I should wear the stockings even when I am sleep c. Every four hours I should remove the stockings for a half hour

d. I should put on the stockings before getting out of bed in the morning TEST V - Care of Clients with Physiologic and Psychosocial Alterations 1. Mr. Marquez reports of losing his job, not being able to sleep at night, and feeling upset with his wifeNurse John responds to the client, You may want to talk about your employment situation in group todayThe Nurse is using which therapeutic technique? a. Observations b. Restating c. Exploring d. Focusing 2. Tony refuses his evening dose of Haloperidol (Haldol), then becomes extremely agitated in the dayroom while other clients are watching television. He begins cursing and throwing furniture. Nurse Oliver first action is to: a. Check the clients medical record for an order for an as-needed I.M. dose of medication for agitation. b. Place the client in full leather restraints. c. Call the attending physician and report the behavior. d. Remove all other clients from the dayroom. 3. Tina who is manic, but not yet on medication, comes to the drug treatment center. The nurse would not let this client join the group session because: a. The client is disruptive. b. The client is harmful to self.

c. The client is harmful to others. d. The client needs to be on medication first. 4. Dervid, an adolescent boy was admitted for substance abuse and hallucinationsThe clients mother asks Nurse Armando to talk with his husband when he arrives at the hospital. The mother says that she is afraid of what the father might say to the boy. The most appropriate nursing intervention would be to: a. Inform the mother that she and the father can work through this problem themselves. b. Refer the mother to the hospital social worker. c. Agree to talk with the mother and the father together. d. Suggest that the father and son work things out. 5. What is Nurse John likely to note in a male client being admitted for alcohol withdrawal? a. Perceptual disorders. b. Impending coma. c. Recent alcohol intake. d. Depression with mutism. 6. Aira has taken amitriptyline HCL (Elavil) for 3 days, but now complains that it doesnt help and refuses to take it. What should the nurse say or do? a. Withhold the drug. b. Record the clients response c. Encourage the client to tell the doctor. d. Suggest that it takes a while before seeing the results. 7. Dervid, an adolescent has a history of truancy from school, running away from home and barrowingother peoples things without their

permission. The adolescent denies stealing, rationalizing instead that as long as no one was using the items, it was all right to borrow them. It is important for the nurse to understand the psychodynamically, this behavior may be largely attributed to a developmental defect related to the: a. Id b. Ego c. Superego d. Oedipal complex 8. In preparing a female client for electroconvulsive therapy (ECT), Nurse Michelle knows that succinylcoline (Anectine) will be administered for which therapeutic effect? a. Short-acting anesthesia b. Decreased oral and respiratory secretions. c. Skeletal muscle paralysis. d. Analgesia. 9. Nurse Gina is aware that the dietary implications for a client in manic phase of bipolar disorder is: a. Serve the client a bowl of soup, buttered French bread, and apple slices. b. Increase calories, decrease fat, and decrease protein. c. Give the client pieces of cut-up steak, carrots, and an apple. d. Increase calories, carbohydrates, and protein. 10. What parental behavior toward a child during an admission procedure should cause Nurse Ron to suspect child abuse? a. Flat affect

b. Expressing guilt c. Acting overly solicitous toward the child. d. Ignoring the child. 11. Nurse Lynnette notices that a female client with obsessive-compulsive disorder washes her hands for long periods each day. How should the nurse respond to this compulsive behavior? a. By designating times during which the client can focus on the behavior. b. By urging the client to reduce the frequency of the behavior as rapidly as possible. c. By calling attention to or attempting to prevent the behavior. d. By discouraging the client from verbalizing anxieties. 12. After seeking help at an outpatient mental health clinic, Ruby who was raped while walking her dog is diagnosed with posttraumatic stress disorder (PTSD). Three months later, Ruby returns to the clinic, complaining of fear, loss of control, and helpless feelings. Which nursing intervention is most appropriate for Ruby? a. Recommending a high-protein, low-fat diet. b. Giving sleep medication, as prescribed, to restore a normal sleep- wake cycle. c. Allowing the client time to heal. d. Exploring the meaning of the traumatic event with the client. 13. Meryl, age 19, is highly dependent on her parents and fears leaving home to go away to college. Shortly before the semester starts, she complains that her legs are paralyzed and is

rushed to the emergency department. When physical examination rules out a physical cause for her paralysis, the physician admits her to the psychiatric unit where she is diagnosed with conversion disorder. Meryl asks the nurse, "Why has this happened to me?" What is the nurse's best response? a. "You've developed this paralysis so you can stay with your parents. You must deal with this conflict if you want to walk again." b. "It must be awful not to be able to move your legs. You may feel better if you realize the problem is psychological, not physical." c. "Your problem is real but there is no physical basis for it. We'll work on what is going on in your life to find out why it's happened." d. "It isn't uncommon for someone with your personality to develop a conversion disorder during times of stress." 14. Nurse Krina knows that the following drugs have been known to be effective in treating obsessive-compulsive disorder (OCD): a. benztropine (Cogentin) and diphenhydramine (Benadryl). b. chlordiazepoxide (Librium) and diazepam (Valium) c. fluvoxamine (Luvox) and clomipramine (Anafranil) d. divalproex (Depakote) and lithium (Lithobid) 15. Alfred was newly diagnosed with anxiety disorder. The physician prescribed buspirone

(BuSpar). The nurse is aware that the teaching instructions for newly prescribed buspirone should include which of the following? a. A warning about the drugs delayed therapeutic effect, which is from 14 to 30 days. b. A warning about the incidence of neuroleptic malignant syndrome (NMS). c. A reminder of the need to schedule blood work in 1 week to check blood levels of the drug. d. A warning that immediate sedation can occur with a resultant drop in pulse. 16. Richard with agoraphobia has been symptom free for 4 months. Classic signs and symptoms of phobias include: a. Insomnia and an inability to concentrate. b. Severe anxiety and fear. c. Depression and weight loss. d. Withdrawal and failure to distinguish reality from fantasy. 17. Which medications have been found to help reduce or eliminate panic attacks? a. Antidepressants b. Anticholinergics c. Antipsychotics d. Mood stabilizers 18. A client seeks care because she feels depressed and has gained weight. To treat her atypical depression, the physician prescribes tranylcypromine sulfate (Parnate), 10 mg by mouth twice per day. When this drug is used to treat atypical depression, what is its onset of action?

a. 1 to 2 days b. 3 to 5 days c. 6 to 8 days d. 10 to 14 days 19. A 65 years old client is in the first stage of Alzheimer's disease. Nurse Patricia should plan to focus this client's care on: a. Offering nourishing finger foods to help maintain the client's nutritional status. b. Providing emotional support and individual counseling. c. Monitoring the client to prevent minor illnesses from turning into major problems. d. Suggesting new activities for the client and family to do together. 20. The nurse is assessing a client who has just been admitted to the emergency department. Which signs would suggest an overdose of an antianxiety agent? a. Combativeness, sweating, and confusion b. Agitation, hyperactivity, and grandiose ideation c. Emotional lability, euphoria, and impaired memory d. Suspiciousness, dilated pupils, and increased blood pressure 21. The nurse is caring for a client diagnosed with antisocial personality disorder. The client has a history of fighting, cruelty to animals, and stealing. Which of the following traits would the nurse be most likely to uncover during assessment? a. History of gainful employment

b. Frequent expression of guilt regarding antisocial behavior c. Demonstrated ability to maintain close, stable relationships d. A low tolerance for frustration 22. Nurse Amy is providing care for a male client undergoing opiate withdrawal. Opiate withdrawal causes severe physical discomfort and can be life-threatening. To minimize these effects, opiate users are commonly detoxified with: a. Barbiturates b. Amphetamines c. Methadone d. Benzodiazepines 23. Nurse Cristina is caring for a client who experiences false sensory perceptions with no basis in reality. These perceptions are known as: a. Delusions b. Hallucinations c. Loose associations d. Neologisms 24. Nurse Marco is developing a plan of care for a client with anorexia nervosa. Which action should the nurse include in the plan? a. Restricts visits with the family and friends until the client begins to eat. b. Provide privacy during meals. c. Set up a strict eating plan for the client. d. Encourage the client to exercise, which will reduce her anxiety. 25. Tim is admitted with a diagnosis of delusions of grandeur. The nurse is aware that this diagnosis

reflects a belief that one is: a. Highly important or famous. b. Being persecuted c. Connected to events unrelated to oneself d. Responsible for the evil in the world. 26. Nurse Jen is caring for a male client with manic depression. The plan of care for a client in a manic state would include: a. Offering a high-calorie meals and strongly encouraging the client to finish all food. b. Insisting that the client remain active through the day so that hell sleep at night. c. Allowing the client to exhibit hyperactive, demanding, manipulative behavior without setting limits. 136 d. Listening attentively with a neutral attitude and avoiding power struggles. 27. Ramon is admitted for detoxification after a cocaine overdose. The client tells the nurse that he frequently uses cocaine but that he can control his use if he chooses. Which coping mechanism is he using? a. Withdrawal b. Logical thinking c. Repression d. Denial 28. Richard is admitted with a diagnosis of schizotypal personality disorder. hich signs would this client exhibit during social situations?

a. Aggressive behavior b. Paranoid thoughts c. Emotional affect d. Independence needs 29. Nurse Mickey is caring for a client diagnosed with bulimia. The most appropriate initial goal for a client diagnosed with bulimia is to: a. Avoid shopping for large amounts of food. b. Control eating impulses. c. Identify anxiety-causing situations d. Eat only three meals per day. 30. Rudolf is admitted for an overdose of amphetamines. When assessing the client, the nurse should expect to see: a. Tension and irritability b. Slow pulse c. Hypotension d. Constipation 31. Nicolas is experiencing hallucinations tells the nurse, The voices are telling me Im no good The client asks if the nurse hears the voices. The most appropriate response by the nurse would be: a. It is the voice of your conscience, which only you can control b. No, I do not hear your voices, but I believe you can hear them c. The voices are coming from within you and only you can hear them d. Oh, the voices are a symptom of your illnessdont pay any attention to them 32. The nurse is aware that the side effect of electroconvulsive therapy that a client may

experience: a. Loss of appetite b. Postural hypotension c. Confusion for a time after treatment d. Complete loss of memory for a time 33. A dying male client gradually moves toward resolution of feelings regarding impending death. Basing care on the theory of Kubler-Ross, Nurse Trish plans to use nonverbal interventions when assessment reveals that the client is in the: a. Anger stage b. Denial stage c. Bargaining stage d. Acceptance stage 34. The outcome that is unrelated to a crisis state is: a. Learning more constructive coping skills b. Decompensation to a lower level of functioning. c. Adaptation and a return to a prior level of functioning. d. A higher level of anxiety continuing for more than 3 months. 35. Miranda a psychiatric client is to be discharged with orders for haloperidol (haldol) therapy. When developing a teaching plan for discharge, the nurse should include cautioning the client against: a. Driving at night b. Staying in the sun c. Ingesting wines and cheeses d. Taking medications containing aspirin 36. Jen a nursing student is anxious about the

upcoming board examination but is able to study intently and does not become distracted by a roommates talking and loud musicThe students ability to ignore distractions and to focus on studying demonstrates: a. Mild-level anxiety b. Panic-level anxiety c. Severe-level anxiety d. Moderate-level anxiety 37. When assessing a premorbid personality characteristic of a client with a major depression, it would be unusual for the nurse to find that this client demonstrated: a. Rigidity b. Stubbornness 137 c. Diverse interest d. Over meticulousness 38. Nurse Krina recognizes that the suicidal risk for depressed client is greatest: a. As their depression begins to improve b. When their depression is most severe c. Before any type of treatment is started d. As they lose interest in the environment 39. Nurse Kate would expect that a client with vascular dementis would experience: a. Loss of remote memory related to anoxia b. Loss of abstract thinking related to emotional state c. Inability to concentrate related to decreased stimuli d. Disturbance in recalling recent events

related to cerebral hypoxia. 40. Josefina is to be discharged on a regimen of lithium carbonate. In the teaching plan for discharge the nurse should include: a. Advising the client to watch the diet carefully b. Suggesting that the client take the pills with milk c. Reminding the client that a CBC must be done once a month. d. Encouraging the client to have blood levels checked as ordered. 41. The psychiatrist orders lithium carbonate 600 mg p.o t.i.d for a female client. Nurse Katrina would be aware that the teachings about the side effects of this drug were understood when the client state, I will call my doctor immediately if I notice any: a. Sensitivity to bright light or sun b. Fine hand tremors or slurred speech c. Sexual dysfunction or breast enlargement d. Inability to urinate or difficulty when urinating 42. Nurse Mylene recognizes that the most important factor necessary for the establishment of trust in a critical care area is: a. Privacy b. Respect c. Empathy d. Presence 43. When establishing an initial nurse-client relationship, Nurse Hazel should explore with the client the:

a. Clients perception of the presenting problem. b. Occurrence of fantasies the client may experience. c. Details of any ritualistic acts carried out by the client d. Clients feelings when externalcontrols are instituted. 44. Tranylcypromine sulfate (Parnate) is prescribed for a depressed client who has not responded to the tricyclic antidepressants. After teaching the client about the medication, Nurse Marian evaluates that learning has occurred when the client states, I will avoid a. Citrus fruit, tuna, and yellow vegetables b. Chocolate milk, aged cheese, and yogurt c. Green leafy vegetables, chicken, and milk d. Whole grains, red meats, and carbonated soda 45. Nurse John is a aware that most crisis situations should resolve in about: a. 1 to 2 weeks b. 4 to 6 weeks c. 4 to 6 months d. 6 to 12 months 46. Nurse Judy knows that statistics show that in adolescent suicide behavior: a. Females use more dramatic methods than males

b. Males account for more attempts than do females c. Females talk more about suicide before attempting it d. Males are more likely to use lethal methods than are females 47. Dervid with paranoid schizophrenia repeatedly uses profanity during an activity therapy session. Which response by the nurse would be most appropriate? a. "Your behavior won't be tolerated. Go to your room immediately." 138 b. "You're just doing this to get back at me for making you come to therapy." c. "Your cursing is interrupting the activity. Take time out in your room for 10 minutes." d. "I'm disappointed in you. You can't control yourself even for a few minutes." 48. Nurse Maureen knows that the nonantipsychotic medication used to treat some clients with schizoaffective disorder is: a. phenelzine (Nardil) b. chlordiazepoxide (Librium) c. lithium carbonate (Lithane) d. imipramine (Tofranil) 49. Which information is most important for the nurse Trinity to include in a teaching plan for a male schizophrenic client taking clozapine (Clozaril)? a. Monthly blood tests will be necessary. b. Report a sore throat or fever to the

physician immediately. c. Blood pressure must be monitored for hypertension. d. Stop the medication when symptoms subside. 50. Ricky with chronic schizophrenia takes neuroleptic medication is admitted to the psychiatric unit. Nursing assessment reveals rigidity, fever, hypertension, and diaphoresis. These findings suggest which life- threatening reaction: a. Tardive dyskinesia. b. Dystonia. c. Neuroleptic malignant syndrome. d. Akathisia. 51. Which nursing intervention would be most appropriate if a male client develop orthostatic hypotension while taking amitriptyline (Elavil)? a. Consulting with the physician about substituting a different type of antidepressant. b. Advising the client to sit up for 1 minute before getting out of bed. c. Instructing the client to double the dosage until the problem resolves. d. Informing the client that this adverse reaction should disappear within 1 week. 52. Mr. Cruz visits the physician's office to seek treatment for depression, feelings of hopelessness, poor appetite, insomnia, fatigue, low self- esteem, poor concentration, and difficulty making decisions. The client states that these symptoms began at least 2 years ago.

Based on this report, the nurse Tyfany suspects: a. Cyclothymic disorder. b. Atypical affective disorder. c. Major depression. d. Dysthymic disorder. 53. After taking an overdose of phenobarbital (Barbita), Mario is admitted to the emergency department. Dr. Trinidad prescribes activated charcoal (Charcocaps) to be administered by mouth immediately. Before administering the dose, the nurse verifies the dosage ordered. What is the usual minimum dose of activated charcoal? a. 5 g mixed in 250 ml of water b. 15 g mixed in 500 ml of water c. 30 g mixed in 250 ml of water d. 60 g mixed in 500 ml of water 54. What herbal medication for depression, widely used in Europe, is now being prescribed in the United States? a. Ginkgo biloba b. Echinacea c. St. John's wort d. Ephedra 55. Cely with manic episodes is taking lithium. Which electrolyte level should the nurse check before administering this medication? a. Clcium b. Sodium c. Chloride d. Potassium 56. Nurse Josefina is caring for a client who has been diagnosed with delirium. Which statement about

delirium is true? a. It's characterized by an acute onset and lasts about 1 month. b. It's characterized by a slowly evolving onset and lasts about 1 week. c. It's characterized by a slowly evolving onset and lasts about 1 month. d. It's characterized by an acute onset and lasts hours to a number of days. 139 57. Edward, a 66 year old client with slight memory impairment and poor concentration is diagnosed with primary degenerative dementia of the Alzheimer's type. Early signs of this dementia include subtle personality changes and withdrawal from social interactions. To assess for progression to the middle stage of Alzheimer's disease, the nurse should observe the client for: a. Occasional irritable outbursts. b. Impaired communication. c. Lack of spontaneity. d. Inability to perform self-care activities. 58. Isabel with a diagnosis of depression is started on imipramine (Tofranil), 75 mg by mouth at bedtime. The nurse should tell the client that: a. This medication may be habit forming and will be discontinued as soon as the client feels better. b. This medication has no serious adverse effects. c. The client should avoid eating such

foods as aged cheeses, yogurt, and chicken livers while taking the medication. d. This medication may initially cause tiredness, which should become less bothersome over time. 59. Kathleen is admitted to the psychiatric clinic for treatment of anorexia nervosa. To promote the client's physical health, the nurse should plan to: a. Severely restrict the client's physical activities. b. Weigh the client daily, after the evening meal. c. Monitor vital signs, serum electrolyte levels, and acid-base balance. d. Instruct the client to keep an accurate record of food and fluid intake. 60. Celia with a history of polysubstance abuse is admitted to the facility. She complains of nausea and vomiting 24 hours after admission. The nurse assesses the client and notes piloerection, pupillary dilation, and lacrimation. The nurse suspects that the client is going through which of the following withdrawals? a. Alcohol withdrawal b. Cannibis withdrawal c. Cocaine withdrawal d. Opioid withdrawal 61. Mr. Garcia, an attorney who throws books and furniture around the office after losing a case is referred to the psychiatric nurse in the law firm's employee assistance program. Nurse Beatriz knows that the client's behavior most likely

represents the use of which defense mechanism? a. Regression b. Projection c. Reaction-formation d. Intellectualization 62. Nurse Anne is caring for a client who has been treated long term with antipsychotic medication. During the assessment, Nurse Anne checks the client for tardive dyskinesia. If tardive dyskinesia is present, Nurse Anne would most likely observe: a. Abnormal movements and involuntary movements of the mouth, tongue, and face. b. Abnormal breathing through the nostrils accompanied by a thrill c. Severe headache, flushing, tremors, and ataxia. d. Severe hypertension, migraine headache, 63. Dennis has a lithium level of 2.4 mEq/L. The nurse immediately would assess the client for which of the following signs or symptoms? a. Weakness b. Diarrhea c. Blurred vision d. Fecal incontinence 64. Nurse Jannah is monitoring a male client who has been placed inrestraints because of violent behavior. Nurse determines that it will be safe to remove the restraints when: a. The client verbalizes the reasons for the

violent behavior. b. The client apologizes and tells the nurse that it will never happen again. c. No acts of aggression have been observed within 1 hour after the release of two of the extremity restraints. d. The administered medication has taken effect. 140 65. Nurse Irish is aware that Ritalin is the drug of choice for a child with ADHD. The side effects of the following may be noted by the nurse: a. Increased attention span and concentration b. Increase in appetite c. Sleepiness and lethargy d. Bradycardia and diarrhea 66. Kitty, a 9 year old child has very limited vocabulary and interaction skills. She has an I.Q. of 45. She is diagnosed to have Mental retardation of this classification: a. Profound b. Mild c. Moderate d. Severe 67. The therapeutic approach in the care of Armand an autistic child include the following EXCEPT: a. Engage in diversionary activities when acting -out b. Provide an atmosphere of acceptance c. Provide safety measures d. Rearrange the environment to activate the child

68. Jeremy is brought to the emergency room by friends who state that he took something an hour ago. He is actively hallucinating, agitated, with irritated nasal septum. a. Heroin b. Cocaine c. LSD d. Marijuana 69. Nurse Pauline is aware that Dementia unlike delirium is characterized by: a. Slurred speech b. Insidious onset c. Clouding of consciousness d. Sensory perceptual change 70. A 35 year old female has intense fear of riding an elevatorShe claims As if I will die insideThe client is suffering from: a. Agoraphobia b. Social phobia c. Claustrophobia d. Xenophobia 71. Nurse Myrna develops a counter-transference reaction. This is evidenced by: a. Revealing personal information to the client b. Focusing on the feelings of the client. c. Confronting the client about discrepancies in verbal or non-verbal behavior d. The client feels angry towards the nurse who resembles his mother. 72. Tristan is on Lithium has suffered from diarrhea

and vomiting. What should the nurse in-charge do first: a. Recognize this as a drug interaction b. Give the client Cogentin c. Reassure the client that these are common side effects of lithium therapy d. Hold the next dose and obtain an order for a stat serum lithium level 73. Nurse Sarah ensures a therapeutic environment for all the client. Which of the following best describes a therapeutic milieu? a. A therapy that rewards adaptive behavior b. A cognitive approach to change behavior c. A living, learning or working environment. d. A permissive and congenial environment 74. Anthony is very hostile toward one of the staff for no apparent reason. He is manifesting: a. Splitting b. Transference c. Countertransference d. Resistance 75. Marielle, 17 years old was sexually attacked while on her way home from school. She is brought to the hospital by her mother. Rape is an example of which type of crisis: a. Situational b. Adventitious c. Developmental d. Internal 76. Nurse Greta is aware that the following is classified as an Axis I disorder by the Diagnosis

and Statistical Manual of Mental Disorders, Text Revision (DSM-IV-TR) is: a. Obesity b. Borderline personality disorder c. Major depression d. Hypertension 77. Katrina, a newly admitted is extremely hostile toward a staff member she has just met, without apparent reason. According to Freudian theory, the nurse should suspect that the client is experiencing which of the following phenomena? a. Intellectualization b. Transference c. Triangulation d. Splitting 78. An 83year-old male client is in extended care facility is anxious most of the time and frequently complains of a number of vague symptoms that interfere with his ability to eat. These symptoms indicate which of the following disorders? a. Conversion disorder b. Hypochondriasis c. Severe anxiety d. Sublimation 79. Charina, a college student who frequently visited the health center during the past year with multiple vague complaints of GI symptoms before course examinations. Although physical causes have been eliminated, the student continues to express her belief that she has a serious illness. These symptoms are typically of which of the following disorders? a. Conversion disorder

b. Depersonalization c. Hypochondriasis d. Somatization disorder 80. Nurse Daisy is aware that the following pharmacologic agents are sedative- hypnotic medication is used to induce sleep for a client experiencing a sleep disorder is: a. Triazolam (Halcion) b. Paroxetine (Paxil)\ c. Fluoxetine (Prozac) d. Risperidone (Risperdal) 81. Aldo, with a somatoform pain disorder may obtain secondary gain. Which of the following statement refers to a secondary gain? a. It brings some stability to the family b. It decreases the preoccupation with the physical illness c. It enables the client to avoid some unpleasant activity d. It promotes emotional support or attention for the client 82. Dervid is diagnosed with panic disorder with agoraphobia is talking with the nurse in-charge about the progress made in treatment. Which of the following statements indicates a positive client response? a. I went to the mall with my friends last Saturday b. Im hyperventilating only when I have a panic attack c. Today I decided that I can stop taking my medication d. Last night I decided to eat more than a bowl of cereal

83. The effectiveness of monoamine oxidase (MAO) inhibitor drug therapy in a client with posttraumatic stress disorder can be demonstrated by which of the following client self -reports? a. Im sleeping better and dont have nightmares b. Im not losing my temper as much c. Ive lost my craving for alcohol d. Ive lost my phobia for water 84. Mark, with a diagnosis of generalized anxiety disorder wants to stop taking his lorazepam (Ativan). Which of the following important facts should nurse Betty discuss with the client about discontinuing the medication? a. Stopping the drug may cause depression b. Stopping the drug increases cognitive abilities c. Stopping the drug decreases sleeping difficulties d. Stopping the drug can cause withdrawal symptoms 85. Jennifer, an adolescent who is depressed and reported by his parents as having difficulty in school is brought to the community mental health center to be evaluated. Which of the following other health problems would the nurse suspect? a. Anxiety disorder b. Behavioral difficulties c. Cognitive impairment d. Labile moods 86. Ricardo, an outpatient in psychiatric facility is diagnosed with dysthymic disorder. Which of the following statement about dysthymic disorder is

true? a. It involves a mood range from moderate depression to hypomania b. It involves a single manic depression c. Its a form of depression that occurs in the fall and winter d. Its a mood disorder similar to major depression but of mild to moderate severity 87. The nurse is aware that the following ways in vascular dementia different from Alzheimers disease is: a. Vascular dementia has more abrupt onset b. The duration of vascular dementia is usually brief c. Personality change is common in vascular dementia d. The inability to perform motor activities occurs in vascular dementia 88. Loretta, a newly admitted client was diagnosed with delirium and has history of hypertension and anxiety. She had been taking digoxin, furosemide (Lasix), and diazepam (Valium) for anxietyThis clients impairment may be related to which of the following conditions? a. Infection b. Metabolic acidosis c. Drug intoxication d. Hepatic encephalopathy 89. Nurse Ron enters a clients room, the client says,Theyre crawling on my sheets! Get them off my bed!Which of the following assessment is the most accurate? a. The client is experiencing aphasia b. The client is experiencing dysarthria

c. The client is experiencing a flight of ideas d. The client is experiencing visual hallucination 90. Which of the following descriptions of a clients experience and behavior can be assessed as an illusion? a. The client tries to hit the nurse when vital signs must be taken b. The client says, I keep hearing a voice telling me to run away c. The client becomes anxious whenever the nurse leaves the bedside d. The client looks at the shadow on a wall and tells the nurse she sees frightening faces on the wall. 91. During conversation of Nurse John with a client, he observes that the client shift from one topic to the next on a regular basis. Which of the following terms describes this disorder? a. Flight of ideas b. Concrete thinking c. Ideas of reference d. Loose association 92. Francis tells the nurse that her coworkers are sabotaging the computer. When the nurse asks questions, the client becomes argumentative. This behavior shows personality traits associated with which of the following personality disorder? a. Antisocial b. Histrionic c. Paranoid d. Schizotypal 93. Which of the following interventions is important for a Cely experiencing with paranoid

personality disorder taking olanzapine (Zyprexa)? a. Explain effects of serotonin syndrome b. Teach the client to watch for extrapyramidal adverse reaction c. Explain that the drug is less affective if the client smokes d. Discuss the need to report paradoxical effects such as euphoria 94. Nurse Alexandra notices other clients on the unit avoiding a client diagnosed with antisocial personality disorder. When discussing appropriate behavior in group therapy, which of the following comments is expected about this client by his peers? a. Lack of honesty b. Belief in superstition c. Show of temper tantrums d. Constant need for attention 95. Tommy, with dependent personality disorder is working to increase his self- esteem. Which of the following statements by the Tommy shows teaching was successful? a. Im not going to look just at the negative things about myself b. Im most concerned about my level of competence and progress c. Im not as envious of the things other people have as I used to be d. I find I cant stop myself from taking over things other should be doing 96. Norma, a 42-year-old client with a diagnosis of chronic undifferentiated schizophrenia lives in a rooming house that has a weekly nursing clinic. She scratches while she tells the nurse she feels

creatures eating away at her skin. Which of the following interventions should be done first? a. Talk about his hallucinations and fears b. Refer him for anticholinergic adverse reactions c. Assess for possible physical problems such as rash d. Call his physician to get his medication increased to control his psychosis 97. Ivy, who is on the psychiatric unit is copying and imitating the movements of her primary nurse. During recovery, she says, I thought the nurse was my mirror. I felt connected only when I saw my nurseThis behavior is known by which of the following terms? a. Modeling b. Echopraxia c. Ego-syntonicity d. Ritualism 98. Jun approaches the nurse and tells that he hears a voice telling him that hes evil and deserves to die. Which of the following terms describes the clients perception? a. Delusion b. Disorganized speech c. Hallucination d. Idea of reference 99. Mike is admitted to a psychiatric unit with a diagnosis of undifferentiated schizophrenia. Which of the following defense mechanisms is probably used by mike? a. Projection b. Rationalization c. Regression

d. Repression 100. Rocky has started taking haloperidol (Haldol). Which of the following instructions is most appropriate for Ricky before taking haloperidol? a. Should report feelings of restlessness or agitation at once b. Use a sunscreen outdoors on a year-round basis c. Be aware youll feel increased energy taking this drug d. This drug will indirectly control essential hypertension.

Вам также может понравиться